Nothing Special   »   [go: up one dir, main page]

Critical Care Examination and Board Review (Jan 9, 2019) - (1259834352) - (McGraw-Hill) 1st Edition Go Full Chapter Free

Download as pdf or txt
Download as pdf or txt
You are on page 1of 58

Critical Care Examination and Board

Review (Jan 9,
2019)_(1259834352)_(McGraw-Hill) 1st
Edition Go
Visit to download the full and correct content document:
https://ebookstep.com/product/critical-care-examination-and-board-review-jan-9-2019
_1259834352_mcgraw-hill-1st-edition-go/
More products digital (pdf, epub, mobi) instant
download maybe you interests ...

Cardiology Board Review - ECG, Hemodynamic and


Angiographic Unknowns (Aug 5,
2019)_(1119423236)_(Wiley-Blackwell) 1st Edition George
A. Stouffer
https://ebookstep.com/product/cardiology-board-review-ecg-
hemodynamic-and-angiographic-unknowns-
aug-5-2019_1119423236_wiley-blackwell-1st-edition-george-a-
stouffer/

Critical Care EEG Basics-Rapid Bedside EEG Reading for


Acute Care Providers (Feb 29,
2024)_(1009261169)_(Cambridge University Press) Jadeja

https://ebookstep.com/product/critical-care-eeg-basics-rapid-
bedside-eeg-reading-for-acute-care-providers-
feb-29-2024_1009261169_cambridge-university-press-jadeja/

Fundamentals of Critical Care-A Textbook for Nursing


and Healthcare Students (Oct 10,
2022)_(1119783259)_(Wiley-Blackwell) 1st Edition Ian
Peate
https://ebookstep.com/product/fundamentals-of-critical-care-a-
textbook-for-nursing-and-healthcare-students-
oct-10-2022_1119783259_wiley-blackwell-1st-edition-ian-peate/

DeGowin's Diagnostic Examination, 11e (July 7,


2020)_(1260134873)_(McGraw Hill) 11th Edition Suneja

https://ebookstep.com/product/degowins-diagnostic-
examination-11e-july-7-2020_1260134873_mcgraw-hill-11th-edition-
suneja/
Critical Care Medicine - An Algorithmic Approach (Apr
15, 2022)_(0323696074)_(Elsevier) 1st Edition Goldfarb-
Rumyantzev

https://ebookstep.com/product/critical-care-medicine-an-
algorithmic-approach-apr-15-2022_0323696074_elsevier-1st-edition-
goldfarb-rumyantzev/

Civilizations and Cultures Translation and Review 5th


Edition Jing Zhao

https://ebookstep.com/product/civilizations-and-cultures-
translation-and-review-5th-edition-jing-zhao/

Fortschritte der Physik Progress of Physics Band 29


Heft 9

https://ebookstep.com/product/fortschritte-der-physik-progress-
of-physics-band-29-heft-9/

A era da conexão contínua (Maio 2019) 9705th Edition


Harvard Business Review Brasil

https://ebookstep.com/product/a-era-da-conexao-continua-
maio-2019-9705th-edition-harvard-business-review-brasil/

Populism and The People in Contemporary Critical


Thought 1st Edition Susan Miller

https://ebookstep.com/product/populism-and-the-people-in-
contemporary-critical-thought-1st-edition-susan-miller/
McGRAW-HILL EDUCATION
SPECIALTY BOARD REVIEW

Examination and Board Review

• Covers all key topics on the ABIM


Critical Care exam blueprint

• Offers multispecialty perspective


to critical care issues

• Includes the latest evidence-based


guidelines in critical care medicine

• More than 500 questions with detailed


answer explanations

• Numerous high-quality images,


including: x-rays, CT scans, ultrasound,
and electrocardiograms

RONALDO COLLO GO, MD


Critical Care Examination
and Board Review

Go_FM_pi-xvi.indd 1 9/11/18 5:47 PM


Critical Care Examination
and Board Review

Ronaldo Collo Go, MD


Faculty
Division of Pulmonary, Critical Care, and Sleep Medicine
Mount Sinai Beth Israel
New York, New York
Division of Pulmonary, Critical Care, and Sleep Medicine
Crystal Run Health Care
Middletown, New York

New York Chicago San Francisco Athens London Madrid Mexico City
Milan New Delhi Singapore Sydney Toronto

Go_FM_pi-xvi.indd 3 9/11/18 5:47 PM


Copyright © 2019 by McGraw-Hill Education. All rights reserved. Except as permitted under the United States Copyright Act of 1976,
no part of this publication may be reproduced or distributed in any form or by any means, or stored in a database or retrieval system,
without the prior written permission of the publisher.

ISBN: 978-1-25-983436-3
MHID: 1-25-983436-0

The material in this eBook also appears in the print version of this title: ISBN: 978-1-25-983435-6,
MHID: 1-25-983435-2.

eBook conversion by codeMantra


Version 1.0

All trademarks are trademarks of their respective owners. Rather than put a trademark symbol after every occurrence of a trademarked
name, we use names in an editorial fashion only, and to the benefit of the trademark owner, with no intention of infringement of the
trademark. Where such designations appear in this book, they have been printed with initial caps.

McGraw-Hill Education eBooks are available at special quantity discounts to use as premiums and sales promotions or for use in corpo-
rate training programs. To contact a representative, please visit the Contact Us page at www.mhprofessional.com.

TERMS OF USE

This is a copyrighted work and McGraw-Hill Education and its licensors reserve all rights in and to the work. Use of this work is subject
to these terms. Except as permitted under the Copyright Act of 1976 and the right to store and retrieve one copy of the work, you may
not decompile, disassemble, reverse engineer, reproduce, modify, create derivative works based upon, transmit, distribute, disseminate,
sell, publish or sublicense the work or any part of it without McGraw-Hill Education’s prior consent. You may use the work for your
own noncommercial and personal use; any other use of the work is strictly prohibited. Your right to use the work may be terminated if
you fail to comply with these terms.

THE WORK IS PROVIDED “AS IS.” McGRAW-HILL EDUCATION AND ITS LICENSORS MAKE NO GUARANTEES OR WAR-
RANTIES AS TO THE ACCURACY, ADEQUACY OR COMPLETENESS OF OR RESULTS TO BE OBTAINED FROM USING
THE WORK, INCLUDING ANY INFORMATION THAT CAN BE ACCESSED THROUGH THE WORK VIA HYPERLINK OR
OTHERWISE, AND EXPRESSLY DISCLAIM ANY WARRANTY, EXPRESS OR IMPLIED, INCLUDING BUT NOT LIMITED
TO IMPLIED WARRANTIES OF MERCHANTABILITY OR FITNESS FOR A PARTICULAR PURPOSE. McGraw-Hill Education
and its licensors do not warrant or guarantee that the functions contained in the work will meet your requirements or that its opera-
tion will be uninterrupted or error free. Neither McGraw-Hill Education nor its licensors shall be liable to you or anyone else for any
inaccuracy, error or omission, regardless of cause, in the work or for any damages resulting therefrom. McGraw-Hill Education has no
responsibility for the content of any information accessed through the work. Under no circumstances shall McGraw-Hill Education and/
or its licensors be liable for any indirect, incidental, special, punitive, consequential or similar damages that result from the use of or
inability to use the work, even if any of them has been advised of the possibility of such damages. This limitation of liability shall apply
to any claim or cause whatsoever whether such claim or cause arises in contract, tort or otherwise.
J.M.J.
Jude, Pio, Anthony, Francis, Gerard, Faustina, Michael
Evangeline, Benjamin, Anna, Jean, Juan, Gabby
Go Family, Truyol Family, Bediones Family, Rodriguez Family,
Zapanta Family, Collo Family, Pangan Family
Uncle June, Aunt Rosie, Aunt Leah
Lolo Natalio, Lola Brigida, Lolo Jose, Lola Rosalina, Lola Sabina, Lola Rosie,
Lola Isabel, Lola Susanna, Lola Rosa, Lola Angelita
Aunt Lydia, Aunt Imelda, Uncle Rodolfo, Aunt Letitia
Our patients

Go_FM_pi-xvi.indd 5 9/11/18 5:47 PM


Contents
Contributors ix
Preface xv

1. Acid–Base Disorders 01 13. Airway Management 269


Nicole K. Zagelbaum, DO, MPH, Osmaan Minhas, Joseph Cerminara, MD, Timothy Quinn, MD,
DO, Sajid A. Mir, MD, and Sergio Obligado, MD Ronaldo Collo Go, MD, and Ananda Dharshan, MD
2. Electrolyte Disorders 23 14. Analgesia, Sedation, Delirium, and Coma 289
Sergio Obligado, MD, Romeo Quilitan, MD, Maria Osundele, PharmD, CGP, BCPS,
Eduardo Pinto, MD, Dulya Santikul, DO, Ronaldo Collo Go, MD, Samer El Zarif, MD,
Blesit George, DO, and Steven Grundfast, MD Aamir Gilani, MD, and Murali G. Krishna, MD
3. Hemodynamic Monitoring 53 15. Stroke 305
Mayanka Tickoo, MD, Ronaldo Collo Go, MD, Mais N. Al-Kawaz, MD, and
and Michael McBrine, MD Alexander E. Merkler, MD
4. Shock 69 16. Status Epilepticus 321
Mayanka Tickoo, MD, Ronaldo Collo Go, MD, Margaret Huynh, DO, and Brandon Foreman, MD
and Michael McBrine, MD
17. Disorders of the Spinal Cord and Peripheral
5. Cardiac Arrhythmias and Hypertensive Nervous System in Critical Care 347
Emergencies 85 Edward W. Bahou, MD, Yaojie Wu, MD,
Candice Kim, MD, Melissa Dakkak, DO, and Shanna K. Patterson, MD
Ronaldo Collo Go, MD, and Prakash Goutham
Suryanarayana, MD 18. Sepsis 365
Navitha Ramesh, MD, Matthew Frank, MD,
6. Acute Coronary Syndromes 121 Eric Bondarsky, MD, Lina Miyakawa, MD,
Uschi Auguste, MD, and Yumiko Kanei, MD and Ronaldo Collo Go, MD
7. Congestive Heart Failure 157
19. Healthcare-Acquired Infections 403
Srikanth Yandrapalli, MD, Sohaib Tariq, MD,
Matthew Frank, MD, Navitha Ramesh, MD,
and Gregg M. Lanier, MD
and Ronaldo Collo Go, MD
8. Hypoxemic Respiratory Failure 173
20. The Immune System and Infection 423
Ronaldo Collo Go, MD, Maureen Dziura, MD,
Ronaldo Collo Go, MD, Navitha Ramesh, MD,
Hala El Chami, MD, and Imrana Qawi, MD
Riffat Mannan, MD, Songyang Yuan, MD,
9. Hypercarbic Respiratory Failure 185 and Mikyung Lee, MD
Sidney Braman, MD, and Steven H. Feinsilver, MD
21. Antimicrobials 463
10. Thromboembolic Disease 199 Diana Gritsenko, PharmD, Marianna Fedorenko,
Ronaldo Collo Go, MD PharmD, BCPS, Navitha Ramesh, MD,
and Anousheh Ghezel-Ayagh, MD
11. Acute Respiratory Distress Syndrome 213
Oleg Epelbaum, MD, and Christian Becker, MD 22. Endocrinology 485
Allison Ann Froehlich, MD
12. Mechanical Ventilation 235
Rania Esteitie, MD, Ronaldo Collo Go, MD, 23. Hematology and Oncology 503
and Maher Tabba, MD Ronaldo Collo Go, MD, and Michael H. Kroll, MD

vii

Go_FM_pi-xvi.indd 7 9/11/18 5:47 PM


viii   Contents

24. Gastroenterology 525 31. Critical Care Ultrasound 673


James F. Crismale, MD, and Thomas Schiano, MD Jason Filopei, MD, Young Im Lee, MD,
and Samuel Acquah, MD
25. Obstetrics 561
Anna Collo Go, MD, Ronaldo Collo Go, MD, 32. Nutrition 693
and Evangeline Collo Go, MD Ronaldo Collo Go, MD
26. Acute Kidney Injury and Renal Replacement 33. Extracorporeal Life Support 707
Therapy 585 Raghad Hussein, MD, Arvind Sundaram, MD,
Saad A. Bhatti, MD, Karen Braich, MD, and Jason A. Stamm, MD
and Vijay Lapsia, MD
34. Hypothermia and Hyperthermia 721
27. Surgical Intensive Care Unit 599 Ronaldo Collo Go, MD
Khalid Sherani, MD, Jennifer Cabot, MD,
35. Biostatistics 727
and Stephen M. Pastores, MD
Kartik Ramakrishna, MD
28. Transplantation 613
36. Ethics, Death, and Organ Donation 737
Sakshi Dua, MD
Chika Nwulu, MD, and Ronaldo Collo Go, MD
29. Trauma and Warfare 625
Appendix 751
Ronaldo Collo Go, MD
30. Toxicology 661 Index 753
Ronaldo Collo Go, MD, Rania Esteitie, MD,
Faisal Tamimi, MD, Han Yu, MD, and Ronni Levy, MD

Go_FM_pi-xvi.indd 8 9/11/18 5:47 PM


Contributors

Samuel Acquah, MD Eric Bondarsky, MD


Director Assistant Director of Critical Care
Medical Intensive Care Unit NYU Langone Orthopedic Hospital
Division of Pulmonary, Critical Care, and Sleep Medicine Division of Pulmonary, Critical Care, and Sleep Medicine
Mount Sinai Hospital New York, New York
New York, New York
Karen Braich, MD
Mais N. Al-Kawaz, MD
Fellow
Resident Division of Nephrology
Department of Neurology Icahn School of Medicine at Mount Sinai
Weill Cornell Medicine Mount Sinai Hospital
New York, New York New York, New York

Uschi Auguste, MD
Sidney Braman, MD
Fellow
Professor of Medicine
Division of Cardiology
Director of Pulmonary Disease Management
Mount Sinai Beth Israel
Icahn School of Medicine at Mount Sinai
New York, New York
Mount Sinai Hospital
New York, New York
Edward W. Bahou, MD
Clinical Neurophysiology Fellow
Jennifer Cabot, MD
Mount Sinai Hospital
New York, New York Fellow
Department of Anesthesiology and Critical Care Medicine
Memorial Sloan Kettering Cancer Center
Christian Becker, MD, Ph.D
New York, New York
Associate Medical Director, eHealth Center
Director, Research & Quality
Westchester Medical Center Health Network Joseph Cerminara, MD
Associate Professor Resident
New York Medical College Department of Anesthesiology
Department of Medicine Roswell Park Cancer Institute
Division of Pulmonary, Critical Care, and Sleep Medicine Buffalo, New York
Valhalla, New York
Hala El Chami, MD
Saad A. Bhatti, MD Fellow
Surgical Trauma ICU Division of Pulmonary, Critical Care, and Sleep Medicine
Elmhurst Hospital Tufts Medical Center
Queens, New York Boston, Massachusetts

ix

Go_FM_pi-xvi.indd 9 9/11/18 5:47 PM


x   Contributors

James F. Crismale, MD Marianna Fedorenko, PharmD, BCPS


Transplant Hepatology/Gastroenterology Fellow Clinical Pharmacy Specialist, Infectious Diseases
Icahn School of Medicine at Mount Sinai Department of Pharmacy
New York, New York Mount Sinai Beth Israel
New York, New York
Melissa Dakkak, DO
Cardiology Fellow Steven H. Feinsilver, MD
University of Arizona School of Medicine Director, Center for Sleep Medicine
Division of Cardiology Division of Pulmonary Medicine
Banner University Medical Center Tucson Lenox Hill Hospital
Tucson, Arizona Professor of Medicine
Hofstra Northwell School of Medicine
Ananda Dharshan, MD New York, New York
Clinical Assistant Professor
Jason Filopei, MD
Assistant Professor of Oncology
Department of Anesthesiology Assistant Professor of Medicine
Jacobs School of Medicine and Biomedical Sciences Pulmonary, Critical Care, and Sleep Medicine, Icahn School
State University of New York at Buffalo of Medicine
Buffalo, New York Assistant Director
and Medical Intensive Care Unit
Associate Director of Intensive Care Unit Mount Sinai Beth Israel
Roswell Park Cancer Institute New York, New York
Buffalo, New York
Brandon Foreman, MD
Sakshi Dua, MD Assistant Clinical Professor of Neurology & Rehabilitation
Medicine
Pulmonary and Critical Care Medicine Fellowship Program
Associate Director for Neurocritical Care Research
Director
Division of Neurocritical Care
Division of Pulmonary, Critical Care, and Sleep Medicine
University of Cincinnati
Assistant Professor of Medicine
231 Albert Sabin Way
Icahn School of Medicine at Mount Sinai
Cincinnati, Ohio
Mount Sinai Hospital
New York, New York
Matthew Frank, MD
Resident
Maureen Dziura, MD
Department of Internal Medicine
Fellow Mount Sinai Beth Israel
Division of Pulmonary, Critical Care, and Sleep Medicine New York, New York
Tufts Medical Center
Boston, Massachusetts Allison Ann Froehlich, MD
Department of Endocrinology
Oleg Epelbaum, MD Lehigh Valley Hospital
Division of Pulmonary, Critical Care, and Sleep Medicine Stroudsburg, Pennsylvania
Westchester Medical Center
Assistant Professor Blesit George, DO
New York Medical College Department of Emergency Medicine
Valhalla, New York Orange Regional Medical Center
Middletown, New York
Rania Esteitie, MD
Assistant Professor of Medicine Anousheh Ghezel-Ayagh, MD
Tufts University School of Medicine Department of Infectious Disease
Division of Pulmonary, Critical Care, and Sleep Medicine Crystal Run Health Care
Boston, Massachusetts Middletown, New York

Go_FM_pi-xvi.indd 10 9/11/18 5:47 PM


Contributors   xi

Aamir Gilani, MD Candice Kim, MD


Division of Pulmonary, Critical Care, and Sleep Medicine Cardiology Fellow
Orange Regional Medical Center University of Arizona School of Medicine
Middletown, New York Division of Cardiology
Banner University Medical Center Tucson
Anna Collo Go, MD Tucson, Arizona
Gullas Medical School
Cebu, Philippines Murali G. Krishna, MD
Division of Pulmonary, Critical Care, and Sleep Medicine
Evangeline Collo Go, MD Orange Regional Medical Center
Middletown, New York
Obstetrics and Gynecology (retired)
Michael H. Kroll, MD
Ronaldo Collo Go, MD, FCCP
Professor of Medicine
Faculty Chief of the Section of Benign Hematology
Division of Pulmonary, Critical Care, and Sleep Medicine UT MD Anderson Cancer Center
Mount Sinai Beth Israel Houston, Texas
New York, New York
Division of Pulmonary, Critical Care, and Sleep Medicine Gregg M. Lanier, MD
Crystal Run Health Care
Director of Pulmonary Hypertension, Associate Director
Middletown, New York
of Heart Failure
Assistant Professor of Medicine, New York Medical College
Diana Gritsenko, PharmD Westchester Medical Center
Multispecialty Clinical Pharmacist Valhalla, New York
Department of Pharmacy
Yale-New Haven Vijay Lapsia, MD
New Haven, Connecticut Assistant Professor of Medicine
Medical Director, Mount Sinai Kidney Center
Steven Grundfast, MD Icahn School of Medicine at Mount Sinai
Division of Pulmonary, Critical Care, and Sleep Medicine Mount Sinai Hospital
Crystal Run Healthcare New York, New York
Middletown, New York
Mikyung Lee, MD
Raghad Hussein, MD Director of Infectious Disease Fellowship Program
Associate Associate Professor of Medicine
Critical Care Medicine Icahn School of Medicine at Mount Sinai
Geisinger Medical Center Mount Sinai Hospital
Danville, Pennsylvania New York, New York

Margaret Huynh, DO Young Im Lee, MD


Department of Neurology & Rehabilitation Medicine Assistant Professor of Medicine
University of Cincinnati Gardner Neuroscience Institute Pulmonary, Critical Care, and Sleep Medicine, Icahn School
University of Cincinnati of Medicine
Cincinnati Ohio Director
Medical Intensive Care Unit
Yumiko Kanei, MD Mount Sinai Beth Israel
New York, New York
Program Director
Interventional Cardiology
Ronni Levy, MD
Division of Cardiology
Mount Sinai Beth Israel Division of Pulmonary, Critical Care, and Sleep Medicine
New York, New York Crystal Run Healthcare
Middletown, New York

Go_FM_pi-xvi.indd 11 9/11/18 5:47 PM


xii   Contributors

Riffat Mannan, MD Stephen M. Pastores, MD


Assistant Professor of Pathology Professor of Medicine and Anesthesiology, Weill Cornell
University of Pennsylvania Medical College of Cornell University
Philadelphia Program Director, Critical Care Medicine
Vice-Chair of Education
Michael McBrine, MD Department of Anesthesiology and Critical Care Medicine
Assistant Professor of Medicine Memorial Sloan Kettering Cancer Center
Tufts Medical School New York, New York
Division of Pulmonary, Critical Care, and Sleep Medicine
Boston, Massachusetts Shanna K. Patterson, MD
Assistant Professor of Neurology
Alexander E. Merkler, MD
Director, Electromyography Laboratory
Assistant Professor of Neurology Mount Sinai West and St. Luke’s Hospitals
Department of Neurology/Neurocritical Care Associate Director, Neurology Residency
Weill Cornell Medicine Mount Sinai Hospital
New York, New York New York, New York
Osmaan Minhas, DO
Eduardo Pinto, MD
Resident
Department of Family Medicine Department of Internal Medicine
Orange Regional Medical Center Crystal Run Healthcare
Middletown, New York Middletown, New York

Sajid A. Mir, MD Imrana Qawi, MD


Program Director Assistant Professor of Medicine
Department of Internal Medicine Tufts School of Medicine
Adjunct Clinical Professor Division of Pulmonary, Critical Care,
Touro Medical College and Sleep Medicine
Orange Regional Medical Center Tufts Medical Center
Middletown, New York Boston, Massachusetts

Lina Miyakawa, MD Romeo Quilitan, MD


Assistant Professor Department of Internal Medicine
Icahn School of Medicine Crystal Run Healthcare
Division of Pulmonary, Critical Care, and Sleep Medicine Middletown, New York
Mount Sinai Beth Israel
New York, New York Timothy Quinn, MD
Chika Nwulu, MD Clinical Assistant Professor
Assistant Professor of Oncology
Division of Internal Medicine
Department of Anesthesiology, Critical Care, and
Crystal Run Healthcare
Pain Medicine
Middletown, New York
Jacobs School of Medicine and Biomedical Sciences
Sergio Obligado, MD State University of New York at Buffalo
and
Adjunct Clinical Professor
Roswell Park Cancer Institute
Touro Medical College
Buffalo, New York
Department of Nephrology
Orange Regional Medical Center
Middletown, New York Kartik Ramakrishna, MD
Assistant Professor
Maria Osundele, PharmD, CGP, BCPS Division of Pulmonary, Critical Care, and Sleep Medicine
Clinical Pharmacy Manager Drexel University College of Medicine
Orange Regional Medical Center Philadelphia
Middletown, New York

Go_FM_pi-xvi.indd 12 9/11/18 5:47 PM


Contributors   xiii

Navitha Ramesh, MD Maher Tabba, MD


Division of Pulmonary, Critical Care, and Sleep Medicine Fellowship Director
Geisinger Wyoming Valley Medical Center Assistant Professor of Medicine
Wilkes-Barre, Pennsylvania Tufts University School of Medicine
Division of Pulmonary, Critical Care, and Sleep Medicine
Dulya Santikul, DO Boston, Massachusetts
Program Director
Department of Internal Medicine Faisal Tamimi, MD
Adjunct Clinical Professor Fellow
Touro Medical College Division of Pulmonary, Critical Care, and Sleep Medicine
Orange Regional Medical Center Tufts Medical Center
Middletown, New York Boston, Massachusetts

Thomas Schiano, MD Sohaib Tariq, MD


Medical Director, Adult Liver Transplantation Fellow
Director of Clinical Hepatology Division of Cardiology
Director, Intestinal Transplantation Westchester Medical Center
Recanati/Miller Transplantation Institute Valhalla, New York
Division of Liver Diseases
Professor of Medicine Mayanka Tickoo, MD
Icahn School of Medicine Fellow
Mount Sinai Medical Center Division of Pulmonary, Critical Care, and Sleep Medicine
New York, New York Tufts Medical Center
Boston, Massachusetts
Khalid Sherani, MD
Fellow Yaojie Wu, MD
Department of Anesthesiology and Critical Care Medicine Clinical Neurophysiology Fellow
Memorial Sloan Kettering Cancer Center Mount Sinai Hospital
New York, New York New York, New York

Jason A. Stamm, MD Srikanth Yandrapalli, MD


Clinical Associate Professor of Medicine Chief Resident
Lewis Katz School of Medicine at Temple University Department of Internal Medicine
and Westchester Medical Center
Associate Valhalla, New York
Division of Pulmonary and Critical Care and Sleep Medicine
Geisinger Medical Center
Han Yu MD
Danville, Pennsylvania
Resident
Department of Internal Medicine
Arvind Sundaram, MD
New York Presbyterian – Queens
Associate New York, New York
Critical Care Medicine
Geisinger Medical Center
Songyang Yuan, MD
Danville, Pennsylvania
Assistant Professor of Medicine
Icahn School of Medicine at Mount Sinai
Prakash Goutham Suryanarayana, MD
Department of Pathology
Assistant Professor of Medicine Mount Sinai Hospital
University of Arizona School of Medicine New York, New York
Division of Cardiology
Banner University Medical Center Tucson
Tucson, Arizona

Go_FM_pi-xvi.indd 13 9/11/18 5:47 PM


xiv   Contributors

Nicole K. Zagelbaum, DO, MPH Samer El Zarif, MD


Resident Division of Pulmonary, Critical Care, and
Department of Internal Medicine Sleep Medicine
Westchester Medical Center Orange Regional Medical Center
Valhalla, New York Middletown, New York

Go_FM_pi-xvi.indd 14 9/11/18 5:47 PM


Preface

Discernment of critical care medicine is derived from multiple Current guidelines from various specialties are incorporated
factors: an understanding of the basics of medicine, access to including their levels and/or grades of recommendation.
the most current evidence, clinical experience, and openness Nomenclature on the stratification of quality of evidence and
to palliative care. Based on these factors, I wanted to create a categories of recommendation vary per country and guideline
one stop reference for critical care. but generally are similar. This book is ideal for the critical care
McGraw-Hill’s Critical Care Examination and Board fellow or intensivist studying for the critical care boards, medi-
Review is an evidence-based multidisciplinary perspective to cal student, resident, or any other healthcare provider inter-
critical care medicine. The format of each chapter is text ested in critical care.
material followed by questions and answers. Authors are It was an amazing journey working on this book and
from major academic centers discussing not only the basic I hope it will strengthen your fund of knowledge to help you
principles in their field but also the most current studies. pass your critical care boards.

xv

Go_FM_pi-xvi.indd 15 9/11/18 5:47 PM


1
C H A P T E R

Acid–Base Disorders
Nicole K. Zagelbaum, DO, MPH, Osmaan Minhas, DO,
Sajid A. Mir, MD, and Sergio Obligado, MD

INTRODUCTION where Paco2 is the partial pressure of CO2 in arterial blood.


The amount of CO2 dissolved in solution is proportional to
The human body needs to regulate free hydrogen ions (H+) the partial pressure of CO2(Pco2), which is in equilibrium
within a narrow window in order to maintain proper protein with Paco2 of alveolar air. The solubility constant of CO2 is
structure and function. In the process of metabolizing carbo- 0.03.2
hydrates, proteins, and fats, 15,000 mmol of volatile acid, car- The effectiveness of the buffering system can be appre-
bon dioxide (CO2), is generated and another 50 to 100 mEq ciated when studying this equation. One measure used
of nonvolatile acid is also produced.1 Despite this, free hydro- to quantify strength of acid in solution is the dissociation
gen ion concentration is maintained at level of 40 nmol/L constant, or pKa. In general, a weak acid will be the most
(10−6 mmol/L).2 When describing hydrogen concentration of effective buffer when pH is within 1 log of its pKa. Bicar-
physiologic solutions, we refer to the pH of the solution rather bonate is not an excellent buffer at physiological pH (its pKa
than the actual concentration. The pH of a solution is defined is only 6.1). However, because CO2 and bicarbonate can be
by the following equation: independently regulated (by the lungs and kidneys, respec-
tively), bicarbonate can be an effective buffer.2 Of course,
pH = − log [H+] other extracellular buffers such as phosphates and plasma
proteins do contribute to preserving pH; however, they are
The pH compatible with human life is in the range of
not considered to be as important. Intracellular buffers, such
6.8 to 7.8.3 In order to maintain serum pH in this range, the
as hemoglobin, are very important to the initial buffering of
human body requires the following:2
respiratory acidosis and alkalosis, and will be discussed later
• an effective buffer system to prevent wide fluctuations in the chapter.
of pH in response to small additions or subtractions of Although the values change depending on the labora-
acid, tory or reference, generally, acidemia refers to serum pH less
• the ability to excrete volatile acids (CO2) via the lungs, and than 7.35 and alkalemia refers to serum pH more than 7.45.
• the ability to excrete nonvolatile acids (sulfuric acid, phos- We refer to acidosis as the physiologic process that leads to
phoric acid, and ammonium) via the kidneys. an acidemia, which can be either a fall in bicarbonate or a
rise in Paco2. Conversely, alkalosis is due to processes that
The bicarbonate/carbon dioxide buffer system is how
increase bicarbonate or decrease Paco2. Additionally, an
clinicians usually analyze acid–base balance at the bedside.
acid–base disturbance is defined as metabolic if the primary
Carbon dioxide and bicarbonate are easily measured; bicar-
process driving it is due to a change in serum bicarbonate,
bonate is important simply because of its high concentration
and as respiratory if it’s driven by a change in Paco2. Table 1-1
in the blood.
summarizes these processes.
Carbon dioxide effectively becomes an acid when dis-
solved in solution as described in the following reaction:
Renal Role in Acid–Base Homeostasis
CO2 ↔ CO2 + H2O ↔ H2CO3 ↔ H + HCO+
3

Although the lungs can assist in excreting carbonic acid, the 50
Gas Phase Aqueous Phase to 100 mEq of nonvolatile acid (eg, sulfuric acid and phosphoric
acid) that is generated by the body each day must be excreted
The relationship of these reactants can be described by
in the urine. There are 2 steps to excretion of nonvolatile acid:
the Henderson-Hasselbalch equation:4
1. Reabsorption of the entire filtered bicarbonate load
6.1 + log [HCO3−] (mostly by the proximal tubule)
pH =
0.03 × Paco2 2. Secretion of hydrogen by the distal tubule

Go_Ch001_p001-p022.indd 1 9/11/18 5:39 PM


2   CHAPTER 1

TABLE 1-1 Acid–Base Disturbances With Appropriate When there is an increase in serum hydrogen ion con-
Responses. Adapted from Berend et al4 centration due to generation of acid or inability to excrete
acid, the following reaction is driven to the right:2
Acid–Base
Disturbance Appropriate Response
H+ + HCO3− ↔ H2CO3 ↔ CO2 + H2O
Metabolic acidosis pH < 7.35 and [HCO ] < 22 mEq/L
3

This ultimately leads to consumption of the major extracel-


Respiratory compensation Paco2 = 1.5 × [HCO3−] + 8 ± 2 mmHg
lular buffer, bicarbonate. If the etiology of acidosis is due
Metabolic alkalosis pH > 7.45 and [HCO3−] > 26 mEq/L to increased bicarbonate loss (as in diarrhea) the reaction
Respiratory compensation Paco2 = [HCO3−] + 15 mmHg is driven to the left, and H+ is ultimately generated in the
reaction.
Respiratory acidosis pH < 7.35 and Paco2 > 45 mmHg
The normal physiologic response to an acid load is to
Acute metabolic [HCO3−] increases by 1 mEq/L minimize the change in pH via several mechanisms:
compensation (< 24 h) for every 10-mmHg rise in
Paco2 or • extracellular buffering (by serum bicarbonate),
change in pH = 0.008 × (40 − Paco2) • intracellular and bone buffering (by hemoglobin and phos-
Chronic metabolic [HCO3−] increases by 4 mEq/L for phate from bone), and
compensation (> 48 h) every 10-mmHg rise in Paco2 or • respiratory compensation (increase in ventilation to excrete
change in pH = 0.003 × (40 − Paco2) Paco2).
Respiratory alkalosis pH > 7.45 and Pco2 < 35 mmHg
These intracellular and extracellular buffer mechanisms
Acute metabolic [HCO3−] decreases by 2 mEq/L for are particularly important to prevent excessive drops in
compensation (< 24 h) every 10-mmHg fall in Paco2 or serum pH. Up to 60% of acid loads will be taken up by intra-
change in pH = 0.008 × (40 − Paco2)
cellular and bone buffers. For example, if 12 mEq of hydrogen
Chronic metabolic [HCO3−] decreases by 4 mEq/L for ions is added to serum, the serum bicarbonate will only fall by
compensation (> 48 h) every 10-mmHg fall in Paco2 or
change in pH = 0.003 × (40 − Paco2)
a total of 5 mEq because of intracellular and bone buffering.5
The fall in serum pH leads to stimulation of peripheral
HCO3− = bicarbonate; Paco2 = partial pressure of carbon dioxide in arterial blood; and central chemoreceptors, which are located on the ven-
Pco2 = partial pressure of carbon dioxide.
trolateral medullary surface and carotid and aortic bodies,
respectively. This stimulation results in a rise of alveolar
ventilation by increasing tidal volume and respiratory rate.
Although the nephron can pump protons against an elec- The increased respiratory response typically begins within 1
trochemical gradient, the minimum urine pH that it can gener- to 2 hours and results in a fall in Paco2. The fall in Paco2 in
ate is about 4.5 to 5. This pH only corresponds to a proton load response to metabolic acidosis will approximately follow the
of about 0.04 mEq/L, which is far less than the 50 to 100 mEq of Winter equation:
hydrogen that needs to be excreted each day.2 Hence, most
of the free hydrogen that is secreted into the lumen is bound Paco2 = 1.5 × [HCO3−] + 8 ± 2 mmHg
to titratable acids (primarily hydrogen phosphate [HPO42−]
or ammonia [NH3]). On a typical diet, 10 to 40 mEq of H+ is If the Paco2 is higher than what is predicted by the above
excreted bound to titratable acid, and another 30 to 60 mEq is equation, then it suggests an additional respiratory acidosis.
bound to NH3. If more acid needs to be excreted (as in the case Conversely, a Paco2 that is lower than predicted suggests a
of underlying acidosis), then the kidney can generate more NH3 simultaneous respiratory alkalosis.
by metabolizing glutamine in the proximal tubule in order to
buffer that excess H+ in the urine. Anion Gap
Once a metabolic acidosis has been identified and the respi-
METABOLIC ACIDOSIS ratory compensation is calculated, the next step in acid–base
analysis involves calculating the anion gap (AG). The serum
As mentioned above, a pH below 7.35 is considered acide- anion gap is defined as
mia. Metabolic acidosis is defined as a pathological process
that results in decreased bicarbonate ion concentration. If Serum Anion Gap = [Na+] – ([Cl−] + [HCO3−])
serum bicarbonate is less than 22 mEq/L, and the serum pH
Of course, there is no true electrolyte gap, as the law of
is less than 7.35, then a metabolic acidosis is present.4 In
electroneutrality dictates that all cations must be equal to all
general, there are 3 physiologic mechanisms for metabolic
anions. It is inconvenient to measure all the anions in every
acidosis:
patient, particularly given how small in concentration some
1. increased generation of acid (eg, diabetic ketoacidosis), electrolytes are. Since sodium, chloride, and bicarbonate are
2. increased bicarbonate loss (eg, diarrhea), and present in very large concentrations in the serum and can dis-
3. inability to excrete acid (eg, renal tubular acidosis).1 play the largest variability, we can monitor them and identify

Go_Ch001_p001-p022.indd 2 9/11/18 5:39 PM


Acid–Base Disorders   3

UC+ concomitant metabolic alkalosis to be present. Similarly, if the


A–
change in bicarbonate is more than 5 mEq greater than the
Anion gap
UA– change in the anion gap, then we would predict a mixed non-
gap acidosis to be present as well. So, the equation for diabetic
HCO3–
ketoacidosis is the following:

ΔAG − Δ HCO3 = 0 ± 5
Na+
In lactic acidosis, the ratio is 0.6 (ie, for every 1 mEq/L rise
in the anion gap, the serum bicarbonate falls by 0.6 mEq/L).
This difference is probably due to the lower renal clearance
Cl– of lactate as compared to ketoacids. Hence, the formula for
lactic acidosis would be adjusted slightly:5

FIGURE 1-1 Schematic of the anion gap. 0.6 × ΔAG − ΔHCO3 = 0 ± 5

the presence of unmeasured anions (UA−) if the anion gap Causes of High Anion Gap Acidosis
rises. The typical unmeasured ions include lactate, phos-
There is a wide differential diagnosis that has to be considered
phate, citrate, sulfate, and, most importantly, albumin. Accu-
when an elevated anion gap is diagnosed. The most common
mulation of 1 of these UA− causes an increase in the anion
categories are lactic acidosis, ketoacidosis (eg, diabetic, alco-
gap because of the buffering by bicarbonate of hydrogen pro-
holic, starvation induced), uremic acidosis, and drug or toxin
duced by these anions:2
ingestion. A helpful mnemonic to remember the different eti-
H+UA− + NaHCO3 ↔ H2CO3 + Na+ + UA− ologies is GOLD MARRK (Table 1-2).4
↔ H2O + CO2 + Na+ + UA−
Lactic Acidosis
Hence, bicarbonate is consumed when buffering the excess Lactic acid is produced from the metabolism of pyruvic acid
hydrogen, leaving only the sodium and UA−. Chloride con- during the process of anaerobic glycolysis. The digestive tract is
centration remains unchanged in this equation, and therefore, responsible for over 50% of the body’s lactic acid production.4,5
the calculated anion gap will rise. Skeletal muscle, brain tissue, skin, and red blood cells (RBCs)
Albumin accounts for about 75% of the anion gap, so also produce lactate, which is metabolized by both the liver
falls in albumin must be accounted for when the calculation and the kidneys. Causes of lactic acidosis can be categorized
is made.4 The “normal” anion gap in most labs runs in the by subtype (A or B), depending on the mechanism.6 Type A
range of 3 to 12 mEq/L; the typical correction for low albu- lactic acidosis is the result of decreased tissue perfusion or
min is to add 2.5 mEq/L to the calculated anion gap for every decreased oxygen delivery that occurs in shock or carbon
drop of albumin of 1 g/dL (Fig. 1-1). monoxide toxicity. Anaerobic glycolysis is increased in these
It should be noted that an abnormally low anion gap can conditions, resulting in higher levels of lactic acid. In the set-
occur in scenarios where there is an excess of unmeasured cat- ting of shock, reduced perfusion of the liver results in a simul-
ion (UC +), which is commonly seen in multiple myeloma (the taneous decrease in lactate metabolism. Type B lactic acidosis
monoclonal protein is positively charged), lithium toxicity, or occurs when mitochondrial or liver function is impaired.
hypercalcemia. A negative anion gap occurs in bromide or The conversion of lactate to pyruvate requires adequate liver
iodide toxicity because they cause a pseudohyperchloremia.4 and mitochondrial function. If either of these is impaired,
lactic acid may accumulate. Metformin and certain antivi-
Delta/Delta ral medications (such as zidovudine or stavudine) also can
inhibit mitochondrial function. Cyanide toxicity results in
When an anion gap acidosis is identified, the next step is to
ensure that there is not a mixed metabolic acid–base distur-
bance. Because there is a predictable relationship between the
fall in bicarbonate and the increase in the anion gap, the clini- TABLE 1-2 Gold Marrk
cian can identify a concurrent non-gap acidosis or metabolic Glycols: propylene and ethylene
alkalosis by analyzing the ratio of the increase in the anion gap 5-oxoproline (pyroglutamic acid)
compared to the fall in serum bicarbonate. This is referred to L-lactate (standard)
D-lactate (short bowel syndrome)
as the delta-delta.4 For example, in diabetic ketoacidosis, the Methanol
increase in the anion gap compared to the fall in bicarbon- Aspirin
ate should be close to 1 to 1. In diabetic ketoacidosis, if the Renal failure
Rhabdomyolysis
change in the anion gap is significantly greater (5 mEq is the
Ketoacidosis
typical error range) than the fall in bicarbonate, we expect a

Go_Ch001_p001-p022.indd 3 9/11/18 5:39 PM


4   CHAPTER 1

type B lactic acidosis because cyanide binds the final enzyme of nicotinamide adenine dinucleotide phosphate (NADP)
of the mitochondrial cytochrome complex (ie, the electron to NADPH (the reduced form) in the body. In diabetic
transfer chain), interrupting normal mitochondrial oxidative ketoacidosis, the average β-hydroxybutyrate–acetoacetate
phosphorylation.6 ratio is 5 to 2. In alcoholic ketoacidosis, the ratio is 20 to 1.5
Notably, urine dipstick testing measures acetoacetate and
Ketoacidosis β-hydroxybutyrate, whereas blood serum ketone levels regis-
Ketoacidosis occurs when glucose is not available to cells due ter only β-hydroxybutyrate (Fig. 1-2).
to a lack of insulin, glucose depletion, or cellular dysfunction.
The 2 major types of ketoacidosis that are seen in clinical prac- Toxins and Drug Ingestions
tice are diabetic ketoacidosis and alcoholic/starvation ketosis. There are several toxins and drugs that increase levels of
In these conditions, there are 2 mechanisms that result in the endogenous acids upon ingestion. In the case of aspirin (ace-
development of ketoacidosis: tylsalicylic acid), the therapeutic range in the serum is usually
20 to 35 mg/dL. When levels exceed 40 to 50 mg/dL, patients
1. increase in free fatty acid delivery due to increased
will present with signs of intoxication.5 Early clinical symp-
lipolysis, and
toms include tinnitus, vertigo, nausea, vomiting, and diarrhea.
2. change in hepatocyte function so that free fatty acids are
Severe overdose can cause hyperthermia, altered mental sta-
converted to ketoacids and not triglycerides.2,5
tus, coma, and death.8 The major anions that accumulate in
During insulin deficiency (which occurs in type 1 diabetes salicylate poisoning are ketoacids and lactic acid, as salicylate
or in starvation states), fatty acids undergo lipolysis. High concentrations in serum are very small and do not significantly
serum glucagon causes fatty acyl coenzyme A (CoA) molecules contribute to the anion gap.5 Salicylate toxicity stimulates
within hepatocytes to be converted to ketones (acetoacetate respiratory centers in the brainstem, leading to a respiratory
and β-hydroxybutyrate).2,5 These ketones are preferentially alkalosis in addition to the anion gap acidosis.
taken up and oxidized by the brain and kidneys. In patients The mainstay of management of salicylate toxicity
with starvation or alcoholic ketosis, the rate of uptake of includes administration of intravenous sodium bicarbon-
ketones by these organs approximates the rate of generation, ate to alkalinize the serum to a pH of 7.5 to 7.55. Salicylic
and hence, acidosis tends not to be as severe.7 The presence of acid diffuses easily into central nervous system (CNS) tissue,
low levels of insulin in starvation limits ketosis to some degree. whereas salicylate ions can be trapped in alkaline serum and
A more detailed discussion of diabetic ketoacidosis can be seen urine, and excreted.8,9 In a patient who presents with severe
in Chapter 22. neurologic symptoms, renal failure, or fluid overload, hemo-
It should be noted that the major type of ketone syn- dialysis should be initiated. Because salicylate is a small mol-
thesized is dependent on etiology of the patient’s ketoacido- ecule with a small volume of distribution, hemodialysis is
sis. The concentrations of each ketone are based on cellular very effective to enhance elimination of the drug and should
reduction-oxidation (redox) levels, or in other words, the ratio be continued until serum levels are below 20 mg/dL.9

Adipose tissue
Glucagon
Catecholamine Insulin
Glucocorticoids

Fatty Acids

Hepatocyte

Mitochondria

Fatty Acyl CoA

Does not react with


NADH NAD nitroprusside in dipstick!
Acetyl CoA

Acetoacetate β-Hydroxybutyrate

FIGURE 1-2 Ketoacid generation in alcoholic and diabetic ketoacidosis.

Go_Ch001_p001-p022.indd 4 9/11/18 5:39 PM


Acid–Base Disorders   5

Acetaminophen is another common over-the-counter med- ketoacidosis, and diabetic ketoacidosis.4 It should be noted
ication that can cause an elevated gap acidosis in patients who are that the osmolal gap in methanol and ethylene glycol inges-
chronic users and simultaneously malnourished. The metabolic tion may only be elevated for several hours after ingestion
acidosis is secondary to the build-up of pyroglutamic acid.4 before the alcohols are metabolized into their anion forms.
Methanol and ethylene glycol are toxic alcohols available Nevertheless, an osmolal gap greater than 20 has a specificity
in automotive antifreeze and commercial solvents that, when of 85% for ingestion of a toxic alcohol.13
ingested, are metabolized by alcohol dehydrogenase and alde-
hyde dehydrogenase (the enzymes that metabolize alcohol) into Uremic Acidosis
toxic metabolites. Methanol, which is available in a number Uremic acidosis is a complication of advanced renal failure
of commercial preparations and in illicit distillations of alco- that occurs when the kidney is unable to excrete daily dietary
hol (moonshine), is metabolized to formaldehyde and then to acids. When glomerular filtration rate (GFR) begins to fall, the
formic acid.9 Formic acid is extremely toxic to the retina and kidney will increase ammonium (NH4+) excretion to maintain
leads to blindness, coma, and death. Ethylene glycol, com- acid balance. However, once the GFR falls to more significant
mon in antifreeze, gets metabolized to glycolate and oxalate, levels (15–20 cc/min), daily nonvolatile acid generation cannot
which precipitate in the kidney to cause tubular injury and be excreted completely and serum bicarbonate falls to levels
obstruction.2 Treatment of these toxic ingestions involves between 12 and 20 mEq/L.2 Severe acidosis usually does not
aggressive hydration to maximize renal clearance and use occur due to buffering by release of calcium salts from the bone.
of fomepizole, a competitive inhibitor of alcohol dehydroge- This can produce a calcium loss over time that results in osteo-
nase. It is recommended that fomepizole be administered if penia in patients with advanced (stage 4 and 5) chronic kidney
any of the below criteria are met:10,11 disease. There are also several unmeasured anions that accumu-
1. Documented recent history of ingesting methanol or late in this process that increase the anion gap. Sulfates accumu-
ethylene glycol and serum osmolal gap more than 10 late from sulfuric acid, which is generated from the metabolism
2. Strong clinical suspicion of methanol or ethylene glycol of amino acids containing sulfur (methionine, cysteine, homo-
poisoning with 2 of the following: cysteine, and taurine). Other unmeasured anions that accumu-
a. Arterial pH less than 7.3 late due to decreased GFR are phosphate, urate, and hippurate.
b. Serum bicarbonate less than 20 mEq/L
c. Osmol gap more than 10
d. Urinary oxalate crystals present Normal Anion Gap Metabolic Acidosis
Normal anion gap metabolic acidosis is due to loss of bicarbon-
Hemodialysis may be required in cases of severe inges- ate in the gastrointestinal (GI) tract, failure to reabsorb bicarbon-
tions, acidemia, or renal failure.10,11 It should also be noted ate in the proximal tubule, or inability to secrete hydrogen in the
that patients with methanol ingestion should be treated with distal tubule (Table 1-3). The renal causes are collectively termed
folic acid (50 mg every 6 hours), as it assists with metabolism renal tubular acidosis (RTA). In a normal anion gap metabolic
of formic acid to CO2 and water.12 acidosis, bicarbonate decreases relative to chloride in a roughly
Use of the osmolal gap can aid early diagnosis of toxic 1-to-1 ratio. As a result, a commonly used synonym of normal
alcohol ingestions. If a patient with metabolic acidosis and anion gap acidosis is hyperchloremic metabolic acidosis.4,5
an unexplained anion gap presents to the emergency room, The urinary anion gap (UAG) is a useful tool to help
the osmolal gap calculation should be performed to rule out distinguish GI losses of bicarbonate from impaired urinary
a toxic alcohol ingestion. The measured serum osmolality is hydrogen excretion. In patients with acidosis, compensa-
compared to the calculated osmolality to determine if a for- tory increased ammonium excretion is expected. Although
eign substance with a low molecular weight and high osmolal urinary ammonium is not typically measured, we can make
activity is present. Examples of such substances are metha- inferences about its excretion based on the difference between
nol, ethylene glycol, isopropyl alcohol, or propylene glycol. the major cations and anions in the urine: sodium, potas-
The osmolal gap calculation is as follows: sium, and chloride.4,5 Hence, the UAG is defined as
glucose BUN UAG = (Na+ + K+) − Cl−
Calculated Posm = 2[Na + ] + +
18 2.8
[ethanol] (if Present) In metabolic acidosis due to diarrhea, normal renal com-
+ pensation leads to an increase in NH4 + excretion, which would
3.7
generate a negative UAG, since chloride excretion would have
A normal osmolal gap is less than 10 mOsm/L; however, to increase to maintain electroneutrality. In most cases of RTA
there is a wide range in the general population. This variance or advanced renal failure where ammonium excretion cannot
leads to potential problems with using the osmolal gap in a clin- be increased, the UAG will be 0 or positive. It should be noted
ical setting. In addition, other clinical situations may involve that proximal RTA may cause a positive UAG due the excretion
elevated osmolal gaps, specifically, lactic acidosis, alcoholic of another anion other than chloride, namely bicarbonate.5

Go_Ch001_p001-p022.indd 5 9/11/18 5:39 PM


6   CHAPTER 1

TABLE 1-3 Etiologies for Normal Anion Gap lowering of urine pH to less than 5.3, which is necessary to
Metabolic Acidosis excrete excess hydrogen in the form of titratable acid (phosphate
and sulfate) and NH4 +.5 Impaired proton secretion can be seen
Gastrointestinal loss of bicarbonate
• Diarrhea
in autoimmune disorders such as Sjögren syndrome, systemic
• Laxative abuse lupus erythematosus, or rheumatoid arthritis (RA). Ampho-
• Enterocutaneous fistulas tericin B can cause increased membrane permeability and a
• Ureterosigmoidostomy or other urinary diversions subsequent leak of H+ ions back in the serum. Type 1 RTA is
Renal bicarbonate loss associated with urine pH more than 5.5, plasma HCO3 less
• Type 2 (proximal) renal tubular acidosis than 15, and renal stones.
• Fanconi syndrome
Type 2 (proximal) RTA is due to the impairment of
• Wilson disease
• Multiple myeloma, amyloid bicarbonate reabsorption at the proximal tubule. This can be
• Sjögren syndrome caused by genetic disorders such as Wilson disease, multiple
• Toxins: lead, cadmium, mercury myeloma, autoimmune conditions, or carbonic anhydrase
• Medication induced (carbonic anhydrase inhibitors)
• Acetazolamide
inhibitors (acetazolamide or topiramate). The acidosis in this
• Topiramate condition tends to be milder than type 1 RTA and frequently
concurrent Fanconi syndrome can be observed, as other ele-
Impaired renal excretion of hydrogen
• Advanced renal disease ments of proximal tubule function are affected.
• Type 1 (distal) renal tubular acidosis Type 4 (distal or hyperkalemic) RTA is usually due to
• Wilson disease aldosterone deficiency or resistance. The hyperkalemia causes
• Primary hyperparathyroidism impaired NH3 generation, and prevents proper buffering of
• Medullary sponge kidney
• Sjögren syndrome and other autoimmune diseases urinary hydrogen ions. Nonsteroidal anti-inflammatory drugs
• Drugs: amphotericin, ifosfamide, lithium (NSAIDs), angiotensin-converting enzyme (ACE) inhibitors,
• Multiple myeloma beta-blockers, and cyclosporine can also produce this effect.
• Type 4 (hyperkalemic) renal tubular acidosis
Type 4 RTA is common in diabetic kidney disease as a result
• Diabetic kidney disease
• Hypoaldosteronism of hyporeninemic hypoaldosteronism.
• Angiotensin-converting enzyme inhibitors, angiotensin receptor
binders, cyclosporine, nonsteroidal anti-inflammatory drugs, Dilutional Acidosis
spironolactone, heparin
A common cause of iatrogenic hyperchloremic acidosis is
Ingestions or toxins large administration of unbuffered crystalloid solutions (eg,
• Ammonium chloride
• Toluene normal saline).4 This most commonly is seen in surgical and
• Cholestyramine trauma patients, where large amounts of saline solution are
Dilutional acidosis with normal saline given to resuscitate unstable patients. In these cases, the nor-
mal bicarbonate of the serum is diluted down before appropri-
ate renal compensation can take place to excrete supplemental
Gastrointestinal Loss of Bicarbonate ammonium and chloride.
Intestinal fluids tend to be alkaline, and as a result, increased
loss of these fluids in the form of diarrhea, enterocutaneous
Treatment
fistula, or villous adenoma lead to a hyperchloremic acidosis. In general, treatment of metabolic acidosis is aimed at treat-
Laxative abuse causes a non-gap acidosis for the same reason. ing the underlying disorder. For example, in lactic acidosis due
If a patient had a ureterosigmoidostomy after a cystectomy to hypotension or sepsis, appropriate volume resuscitation,
in the treatment of bladder cancer, there is frequently a post- pressors, inotropes, and antibiotics should be administered
renal loss of bicarbonate in the urine due to exchange of chlo- to improve tissue perfusion. Similarly, for diabetic ketoaci-
ride for bicarbonate by the intestinal epithelial cells.5 dosis, intravenous insulin will stop lipolysis and ketogenesis.
The benefits of supplemental bicarbonate therapy to replete
Renal Tubular Acidosis the bicarbonate deficit and increase pH remains controversial.
Severe acidosis decreases myocardial contractility and impairs
Acid handling in the kidney is facilitated by 3 mechanisms:
responsiveness to catecholamines.7,14 Hence, treatment of
1. secretion of hydrogen ions at the distal tubule, severe acidosis is recommended by some experts when pH
2. reabsorption of bicarbonate at the proximal tubule, or falls under 7.2.15 It must be noted that the benefit of this prac-
3. generation of NH3, which buffers the urine by binding tice remains unproven.6 There are 2 potential problems that
hydrogen ions in the filtrate. can occur with aggressive bicarbonate replacement:
Disruption of these mechanisms corresponds with the respec- 1. intracellular acidification due to increase in CO2 genera-
tive type of RTA.5 tion, and
A type 1 (distal) RTA is due to impairment of hydrogen 2. a fall in ionized calcium with a rise in pH, resulting in
ion secretion in the distal collecting tubule. This prevents decreased myocardial contractility.

Go_Ch001_p001-p022.indd 6 9/11/18 5:39 PM


Acid–Base Disorders   7

In patients with renal failure, volume overload may treated with oral bicarbonate therapy, as it may relieve dys-
become an issue when sizeable amounts of sodium bicar- pnea (due to pulmonary compensation), reduce bone buff-
bonate crystalloid solutions are administered. Hemodialysis ering, and potentially reduce progression of chronic kidney
could theoretically be a safer modality to correct acidosis as disease.20,21
it can prevent hypocalcemia, hypervolemia, and intracellular
acidification while removing lactate ions.6 However, a hemo- METABOLIC ALKALOSIS
dynamically unstable patient may be challenging to dialyze
safely. Controlled prospective studies are needed to evaluate Metabolic alkalosis is defined as pH more than 7.42 and bicar-
the potential benefits of dialysis for treatment of lactic aci- bonate more than 26 mmol/L. The typical respiratory response
dosis. Even the choice of resuscitation fluids remains contro- to metabolic alkalosis is an increase in Paco2 by 0.75 mmHg
versial with patients in shock; use of chloride-rich isotonic per 1 mEq/L increase in HCO3−.4,22 Complete respiratory
solutions has been shown to be associated with worsening adjustment occurs in 24 to 36 hours.
acute kidney injury (AKI). However, randomized, prospec- Metabolic alkalosis is a result of either GI losses of
tive trials have not verified a benefit to buffered (bicarbonate- hydrogen and chloride, renal losses of hydrogen and chlo-
added) solutions, as far as improved outcomes.6,16,17 ride, intracellular shifts of hydrogen ions, or contraction
In diabetic ketoacidosis (DKA), it has been proposed alkalosis (Fig. 1-3). Whatever the cause for generation of the
that severe acidosis can affect insulin binding to the insulin metabolic alkalosis, a mechanism for maintenance of the
receptor.18 However, small randomized controlled trials have alkalosis has to be present as well, since the kidneys should
not shown a clear benefit of alkali administration to patients be able to filter excess bicarbonate into the urine relatively
with diabetic ketoacidosis.19 There are theoretical risks of easily.4,22 Usually, the mechanism for maintenance of a high
bicarbonate therapy, such as worsening hypokalemia, slower serum bicarbonate is a reduction in GFR with concomitant
resolution of ketonemia, cerebral edema, and paradoxically increased tubular sodium bicarbonate reabsorption and
worsening cerebral spinal fluid (CSF) acidosis, although none increased distal proton excretion at the distal tubule. Other
of these adverse effects was conclusively demonstrated. At possible etiologies for maintenance of metabolic alkalosis
this time, nonbuffered solutions are recommended as the ini- are hyperaldosteronism (primary or secondary), hypoka-
tial resuscitation solutions. lemia, or chloride depletion.2 In cases of volume depletion
As discussed in prior sections, urinary alkalinization is a and secondary hyperaldosteronism, the fall in GFR leads to
useful treatment to improve toxic alcohol and salicylate clear- decreased proximal tubule sodium delivery to the juxtaglo-
ance. Isotonic buffered crystalloid solution should be used in merular apparatus, which stimulates renin release, which in
these situations (eg, addition of 3 amps or 150 mEq sodium turn causes aldosterone production by the adrenal gland.
bicarbonate to a D5 water solution). Other electrolytes need Hyperaldosteronism causes increased cortical and medul-
to be monitored closely, as alkalization can lead to hypokale- lary hydrogen ion secretion by activating type A intercalated
mia and hypocalcemia. Chronic metabolic acidosis should be cells (Fig. 1-4).4

Metabolic
Alkalosis

Intracellular Contraction
GI losses Renal losses shift of H+ alkalosis

Primary
Vomiting mineralocorticoid Hypokalemia Diuresis
excess

Antacids Medications GI losses

Bartter/Gitelman
syndromes

FIGURE 1-3 Multiple causes of metabolic alkalosis. GI = gastrointestinal.

Go_Ch001_p001-p022.indd 7 9/11/18 5:39 PM


8   CHAPTER 1

+
Aldosterone
H+
ATPase
H+

Cl–

K+ HCO3–

ATPase

H+

Type A intercalated cell

Cortical collecting Peritubular


tubule capillary

FIGURE 1-4 Hydrogen adenosine triphosphatase (ATPase) activity in the type A intercalated cell in distal tubule is enhanced by
aldosterone.

Gastrointestinal Losses Renal Losses


Gastrointestinal fluid and electrolyte losses can significantly Metabolic alkalosis can also occur from urinary loss of hydro-
impact ion concentration and lead to volume depletion. gen due to mineralocorticoid excess, sodium and chloride
Decreased effective circulating volume leads to proximal tubule wasting syndromes such as Bartter and Gitelman syndromes,
retention of water (H2O), sodium (Na+), and HCO3−. Common hypokalemia, diuretic use, and iatrogenic administration.
clinical scenarios resulting in metabolic alkalosis as a result of Renal processes that cause hypokalemia can perpetuate alka-
GI losses are the following: losis due to action of the type A intercalated cell (see Fig. 1-4)
and the principal cell (Fig. 1-5),23 both of which are in the cor-
• Vomiting. Excessive vomiting or nasogastric suction of
tical collecting duct of the nephron.
upper gastric secretions have several effects. Gastric secre-
tions have high concentrations of hydrogen chloride, • Mineralocorticoid excess. Mineralocorticoids act to open
which, when lost, directly leads to alkalosis and hypochlo- sodium channels for increased sodium reabsorption at the
remia. Per the mechanism described above, simultaneous principal cells in the collecting tubule; in order to maintain
volume depletion maintains the alkalosis due to proximal electroneutrality, potassium is passively secreted into the
tubule reabsorption of sodium with bicarbonate and acti- tubule (see Fig. 1-5). The most common cause of mineralo-
vation of the renin-angiotensin system. corticoid excess is primary hyperaldosteronism (including
• Antacids. Antacids are commonly comprised of calcium Conn syndrome and bilateral adrenal hyperplasia), Cush-
carbonate or magnesium hydroxide. When antacid medi- ing syndrome, congenital adrenal hyperplasia, and Liddle
cations are ingested in a large quantity, alkali (CO3− or syndrome.
OH−) is absorbed and can lead to a mild alkalosis. This • Bartter/Gitelman syndromes. Bartter and Gitelman syn-
alkali typically can be excreted, unless a fall in GFR occurs. dromes are autosomal recessive disorders characterized by
If hypercalcemia develops from ingestion of these cal- mutations in nephron transporters that lead to sodium and
cium carbonate salts, the increased calcium levels decrease chloride wasting states. The ensuing volume depletion leads
action at both the antidiuretic hormone (ADH) receptors to activation of the renin-angiotensin aldosterone system
and thick ascending limb Na-K-2Cl cotransporter, leading (RAAS), which ultimately results in hypokalemia and met-
to inappropriate diuresis and subsequent volume deple- abolic alkalosis via similar mechanisms outlined above.
tion. In addition, the associated malabsorption can lead to There are several classifications of Bartter syndrome, all
osmotic diarrhea, further worsening volume depletion and resulting in dysfunction in the thick ascending limb (mimics
ability to excrete excess alkali loads.23 use of a loop diuretic). Gitelman syndrome is caused by a

Go_Ch001_p001-p022.indd 8 9/11/18 5:39 PM


Acid–Base Disorders   9

R
Na+

M-R Aldosterone
AIP

K+ AIP

Na+

K+

Cortical collecting Peritubular


tubule capillary

FIGURE 1-5 Mechanism of aldosterone on principal cells. AIP = aldosterone induced protein; K+ = potassium ion; M-R = mineralocorticoid
receptor; Na+ = sodium ion; R = ribosome.

defect in the thiazide-sensitive Na-Cl cotransporter and is considered chloride “responsive” if urine chloride is less than
also characterized by hypomagnesemia.23 25 mEq/, or “resistant” if urine chloride is more than 40 mEq/L.
• Intracellular H+ Shift. In chronic potassium deficiency, the Chloride-responsive metabolic alkalosis is due to vomiting, vol-
body adjusts to create metabolic alkalosis in several ways. ume depletion, the subsiding of a diuretic medication’s effect,
There is an intracellular exchange of hydrogen for potassium villous adenoma, and congenital chloridorrhea.4 These condi-
in order to maintain electroneutrality. In addition, the kidneys tions respond well to sodium chloride and potassium chloride
compensate by upregulating potassium hydrogen ATPase (see administration. If diuresis is needed, acetazolamide, spironolac-
Fig. 1-4). This leads to an increase in potassium at the expense tone, amiloride, or triamterene can be used. In chloride-resistant
of hydrogen loss, exacerbating metabolic alkalosis. states, which can be due to primary hyperaldosteronism, Bartter
• Contraction Alkalosis. Contraction alkalosis occurs in clini- and Gitelman syndromes, or severe hypokalemia, administration
cal scenarios in which the amount of fluid that is lost is high of sodium chloride and/or potassium chloride will not correct
in chloride and relatively low in bicarbonate. The extracel- the alkalosis. Primary aldosteronism, Liddle syndrome, chronic
lular volume contracts around a constant bicarbonate con- licorice ingestion, and mineralocorticoid excess will have meta-
centration (due to decreased GFR and impaired bicarbonate bolic alkalosis with hypertension. Bartter and Gitelman syn-
excretion).2 The most common causes of contraction alkalosis dromes are associated with hypotension or normotension.
are diuretic use, sweat, and gastric losses, as outlined above.
• Alkali Administration. In certain iatrogenic situations, RESPIRATORY ACIDOSIS
patients may develop metabolic alkalosis if large amounts
of bicarbonate or other alkalis are administered. This may During the metabolism of carbohydrates and fats, the body
be seen in the setting of a cardiac arrest, when sodium generates 15,000 mmol of CO2 that has to be excreted via the
bicarbonate is administered rapidly. In addition, in cases of lungs.2,25 As discussed earlier in the chapter, dissolved CO2
hemorrhagic shock in which large amounts of blood prod- combines with water to form carbonic acid (H2CO3):
ucts are administered (≥ 10 units), metabolic alkalosis can
be seen, due to metabolism of sodium citrate (used as an CO2 + H2O ↔ H2CO3
anticoagulant in the packed cells) to sodium bicarbonate.24 The carbonic acid generated is buffered by intracellular
proteins (primarily hemoglobin [Hgb]) and delivered to the
Diagnosis
lungs as seen in the equation below.
Use of a spot urine chloride can be useful to distinguish among
the potential etiologies of alkalosis. Metabolic alkalosis can be H2CO3 + Hgb− ↔ HHgb + HCO3−

Go_Ch001_p001-p022.indd 9 9/11/18 5:39 PM


10   CHAPTER 1

In the alveoli, the process is reversed: Hgb binds O2 and CSF pH, which is sensed by the central medullary chemore-
releases H+, and CO2 is excreted. ceptors, and in turn, decreases ventilatory drive and worsens
The main stimulus for ventilation is the reduction chronic respiratory acidosis.
of arterial oxygen (Pao2) and an elevation in arterial CO2
(Paco2). The chemosensitive areas of the respiratory center in
the medulla sense cerebral interstitial changes in pH. Paco2 Causes
is the major stimulus to respiration, as very minute changes Respiratory acidosis has several systemic physiologic conse-
in Paco2 can induce changes in minute ventilation; a rise in quences and as a result may present with variable and nonspe-
Paco2 of 1 mmHg can increase minute ventilation by 1 to 4 L.25 cific findings. Psychological symptoms include somnolence,
This contrasts with the response to hypoxemia in that min- psychosis, agitation, and delirium (from hypercarbia to brain
ute ventilation may not increase significantly until Pao2 is less tissue). Respiratory consequences include dyspnea (from
than 60 mmHg.25 CO2 delivery to metabolic chemoreceptors in brainstem and
Respiratory acidosis, defined as pH less than 7.35 and carotid body) and respiratory failure (hypercapnia resulting
Paco2 more than 45, can be acute (< 24 hours) or chronic from decreased respiratory drive). Neurologic manifestations
(> 24 hours). There are 2 phases of metabolic compensation include lethargy and coma.
with any drop in arterial pH due to an elevation in Paco2. The The etiology of respiratory acidosis also can be charac-
initial buffering that occurs is due to intracellular protein buff- terized as central/CNS, airway, parenchymal, neuromuscu-
ering (mostly due to hemoglobin, as discussed above). This lar, and miscellaneous (Fig. 1-6). Central causes result in a
immediate buffering causes a rise of bicarbonate by 1 mEq for depression of the respiratory center through pharmacologic
every 10 mmHg increase in Paco2. Interestingly, although the effects or direct injury. Airway obstruction commonly leads
extracellular buffers assist in buffering an acute increase in to an increase in physiologic dead space and may also present
extracellular hydrogen due to metabolic acidosis, the response with hypoxemia. Neuromuscular disease may lead to limited
to acute rise in Pco2 is not as efficient. This is because serum ventilation capacity as a result of physiologic limitation (sco-
bicarbonate cannot buffer proton that is released from Pco2: liosis/obesity) and/or extreme fatigue of central and accessory
respiratory musculature. In patients requiring mechanical
H2CO3 + HCO3− ↔ H2CO3 + HCO3− ventilation for acute respiratory failure due to pneumonia or
ARDS, a permissive hypercapnia strategy may be utilized. In
Hence, the intracellular buffers (eg, hemoglobin) are the only
these patients, tidal volume is kept low in order to prevent
available buffer to proton released from dissolved Pco2.25
lung injury from barotrauma, which may cause significant
After 48 hours or so, renal compensation results in a
hypoventilation and hypercapnia.
4 mEq increase in bicarbonate for every 10 mmHg increase
in Paco2.1,2 Low serum pH leads to an increase in hydrogen
excretion in the distal tubule to accompany bicarbonate reab- Acute Respiratory Acidosis
sorption in the proximal tubule. It should be noted that due Causes of acute respiratory acidosis are numerous, as dis-
to frequent and aggressive use of diuretics in patients with cussed above. Pneumonia, severe asthma, suppression of the
respiratory acidosis (to treat volume overload), an inappropri- respiratory center after cardiac arrest, and drug overdose are
ate metabolic alkalosis may be evident. This causes a higher common causes in patients without underlying lung disease.

Respiratory
Acidosis

Central Airway Parenchymal Neuromuscular Misc

Drugs
Obstruction Emphysema Poliomyelitis Obesity
(anesthetics,
morphine,
sedatives)
Asthma Pneumoconiosis Kyphoscoliosis Hypoventilation

Stoke
Permissive
Bronchitis Myasthenia
hypercapnia
Infection
Muscular
dystrophies

FIGURE 1-6 Causes of respiratory acidosis.

Go_Ch001_p001-p022.indd 10 9/11/18 5:39 PM


Acid–Base Disorders   11

Obstructive sleep apnea can be considered an acute cause of useful to distinguish extrapulmonary versus interstitial disease.
respiratory acidosis since the rise in CO2 occurs primarily at It compares arterial oxygen with alveolar oxygen pressure in
night, and once in the awake state, improves, hence renal com- ambient air; a large difference, or gradient, suggests diffusion
pensation does not have time to occur completely. defect, ventilation/perfusion (V/Q) mismatch, right-to-left
shunt, or increased O2 extraction. A normal or low A-a gradi-
ent suggests hypoventilation or low partial pressure of inspired
Chronic Respiratory Acidosis oxygen (Pio2).30 The normal value (5–10 mmHg) will change
Chronic respiratory acidosis with concurrent hypercapnia depending on age and fraction of inspired oxygen (Fio2). The
is associated with chronic lung disease, including chronic A-a gradient is seen below. Pao2 is alveolar O2, Pao2 is arterial
obstructive pulmonary disorder (COPD) and cystic fibrosis. O2, Paco2 is arterial CO2, Fio2 is fraction of inspired oxygen
In extremely obese patients with the obesity hypoventilation (21% on room air), and atmospheric pressure is 760 pascals
(Pickwickian) syndrome, the increased weight of the chest (Pa) in conventional units, 101.33 kPa in SI units.
wall leads to increased work of breathing and inspiratory
A-a gradient = Pao2 − Pao2 = [(Fio2) ×
weakness, leading to hypoventilation. In addition, decreased
(Atmospheric Pressure − H2O Pressure)
respiratory responsiveness to increased Paco2 and hypoxemia
− (Paco2/0.8)] − Pao2
has been suggested as playing a role.26,27
It is believed that the respiratory centers become less Normal Gradient Estimate = (Age/4) + 4 or 2.5 + 0.21 × Age
sensitive to CO2 during states of chronic hypercapnia, and
hypoxia becomes the main stimulus to respiration. In addi- Treatment
tion, aggressive use of diuretics to treat edema in cor pulmo-
nale increases serum bicarbonate levels (contraction alkalosis), Treatment of acute respiratory acidosis usually is aimed at
which further increase the serum pH and blunt the expected treatment of underlying disease. If these methods are ineffec-
increase in ventilation stimulated by hypercapnia. In normal tive alone, efforts to increase ventilation using noninvasive or
states, hypoxia does not stimulate severe hyperventilation invasive mechanical ventilation frequently are necessary.
because the fall in Paco2 causes a rise in pH, which suppresses Bicarbonate administration for acute respiratory acido-
medullary-induced ventilation. In chronic CO2-retaining sis can be useful in severe acidemia (pH < 7.15), particularly
in the setting of a patient on a ventilator when tidal volumes
states, ventilation will be enhanced once Po2 falls below 80
need to be minimized to decrease barotrauma associated
because a respiratory alkalosis never develops despite high
with high peak and plateau pressures. This permissive hyper-
CO2 levels in the CSF, and medullary centers will not be
capnia strategy may improve outcomes in patients with acute
suppressed.28
respiratory distress syndrome (ARDS) who are difficult to
oxygenate, and prevent ventilator lung injury.31 Some experts
Diagnosis suggest use of sodium bicarbonate (100 mEq) in a D5W solu-
tion, to be infused for a goal pH of 7.3.32 Potential risks of
As previously outlined, acute respiratory acidosis is a result of sodium bicarbonate administration include worsening intra-
an acute decrease in ventilation or a worsening of alveolar ven- cellular acidemia (because of increased generation of CO2,
tilation in patients who have decreased pulmonary reserve. In which can pass across cell membranes), worsening volume
contrast, chronic respiratory acidosis is the result of an ongo- overload (due to volume of intravenous bicarbonate), and
ing disease process. In patients who present with hypercap- CNS effects (increased intracranial pressure and decrease in
nic respiratory failure, immediate labs and arterial blood gas seizure threshold).25,33
(ABG) should be taken. Also of note is that while ABGs are Sodium bicarbonate therapy is not required for patients
preferred, venous blood gas (VBG) analysis may also be used with chronic respiratory acidosis since renal compensation
but will result in a higher Paco2 and lower Pao2.29 As discussed will usually be adequate to maintain a reasonable (> 7.2)
above, if the pH is significantly below normal, acute respira- systemic pH. Treatment is aimed at therapy for the underly-
tory acidosis is likely present (due to limited intracellular buff- ing disease. Excessive oxygen should be avoided given that
ering). If arterial pH is near normal, then chronic respiratory these patients’ drive for respiration is frequently dependent
acidosis is likely, as the patient has had time for renal com- on hypoxia. Feeding patients lower carbohydrate diets may
pensation to take effect. In acute or chronic respiratory aci- be useful to decrease the respiratory quotient and reduce
dosis, Paco2 is elevated, yet pH remains markedly low despite CO2 production.34 Weight loss should be encouraged in obese
a high serum bicarbonate generated by renal compensatory patients to assist alveolar ventilation. Nocturnal positive pres-
mechanisms. sure ventilation can be considered in some COPD patients as
well as in patients with obstructive sleep apnea.
Of note, it is important to be cautious with potassium
Alveolar–Arterial Gradient wasting diuretics in hypercapnic patients because they could
In the event of hypercapnia and hypoxemia leading to respi- cause a secondary metabolic alkalosis, which would suppress
ratory failure, the alveolar–arterial gradient (A-a gradient) is respiratory drive further. Theoretically, acetazolamide may

Go_Ch001_p001-p022.indd 11 9/11/18 5:39 PM


12   CHAPTER 1

be used to correct this metabolic alkalosis; however, it needs respiratory alkalosis, the renal compensation leads to a 4- to
to be used cautiously, as correction of bicarbonate to a nor- 5-mEq decrease in HCO3- levels for every 10-mmHg decrease
mal level (24–28 mEq) would lead to a severe acidemia for in Paco2.22,23
which a patient with intrinsic lung disease may not be able to There are also several systemic changes that are exac-
compensate.25,35,36 erbated based on the patient’s underlying chronic disease
pathology. Decreased bicarbonate levels lead to cerebral
vasoconstriction, reduced oxygenation of CNS, and subse-
RESPIRATORY ALKALOSIS quent somnolence, mental confusion, dizziness, and leth-
argy. Hypoxia leads to a “left” shift to the oxygen dissociation
Respiratory alkalosis can be identified by a decreased Paco2 curve, decreasing oxygen unloading and increasing risk of
(primary hypocapnia), decreased HCO3−, and a subsequent arrhythmias and cardiovascular events. In addition, respira-
increase in systemic pH. There are several simultaneous meta- tory alkalosis leads to increased anionic charge of albumin,
bolic consequences. Acute alkalosis affects renal clearance of which promotes calcium binding and results in a fall in ion-
electrolytes as well as intracellular shifts of Na+, K+, and PO4−, ized calcium, which can cause paresthesia symptoms.22
causing transient ion imbalances, although these mechanisms
are incompletely understood.35 The fall in CO2 leads to a shift
of H+ from intracellular buffers that causes the early compen- Causes
sation for alkalosis. The equation below is driven to the right Respiratory alkalosis has been noted to be common in
to make up for loss of Paco2 due to hyperventilation: patients admitted to the intensive care unit (ICU), and there-
fore, understanding the etiology is critical in order to treat
H+ + HCO3− ↔ H2CO3 ↔ H2O + Pco2 the underlying disease process. Respiratory hyperventilation
can be caused by acute events, including drug ingestion, pain,
This leads to release of hydrogen from intracellular buffers
CNS stimulants, respiratory stimulants, and environmen-
(hemoglobin and lactic acid):
tal conditions. Chronic alkalosis may reflect an underlying
HBuff → H+ + Buff - disease process, including heart failure, anemia, and hepatic
failure. It is interesting to note that acute hypoxia needs to be
This additional hydrogen will be buffered by extracellular severe to trigger respiratory alkalosis because alkalemic pH
bicarbonate and will reduce bicarbonate by approximately tends to inhibit the respiratory center, which dampens the ini-
2 mEq for every 10-mmHg decrease in Pco2.2 Over several tial hyperventilation. However, in chronic hypoxemic states,
days, the change in Paco2 stimulates renal compensation and because the kidney will compensate for alkalosis by decreas-
prevents the excretion of H+. Typically in respiratory alka- ing proton secretion, a greater degree of hyperventilation can
losis, blood Paco2 ranges from 15 to 40 mmHg.36 In chronic occur, leading to a greater fall in Pco2 (Fig. 1-7).2

Respiratory
Alkalosis

Drugs/ Stimulation of
Central Hypoxemia pulmonary Misc
hormones
receptors

Pain, anxiety, Pregnancy/


High altitude Hepatic failure
psychosis progesterone
Hemothorax

Cerebrovascular
Pneumonia Salicylates Heat exposure
accident
Flail chest

Infection Aspiration Cardiac failure Sepsis


Pulmonary
embolism

Tumor Anemia

Trauma

FIGURE 1-7 Causes of respiratory alkalosis.

Go_Ch001_p001-p022.indd 12 9/11/18 5:39 PM


Acid–Base Disorders   13

QUESTIONS Vital signs: BP 99/43 mmHg, RR 26 breaths/min, HR 107


beats/min, Temp 98.4°F
Somnolent
1. A 45-year-old male painter with past medical history of Dry mucous membranes
drug abuse presents to the emergency department with Lungs clear
slurred speech and difficulty walking. His symptoms came Tachycardic, no murmurs
on when he was on the job painting in the basement of a Abdomen soft and nontender
house. No edema, cyanosis, or clubbing
Vital signs: BP 88/43 mmHg, RR 26 breaths/min, HR 100
Arterial Blood Gas:
beats/min, Temp 99°F
pH 6.724
Lethargic but arousable with diffuse muscle weakness
Pco2  20 mmHg
Dry mucous membranes
Lungs clear Labs:
Regular rate and rhythm, no murmurs BUN 56 mg/dL
Abdomen soft and nontender Creatinine  2.4 mg/dL
Sodium 134 mEq/L
Labs: Potassium 5.2 mEq/L
WBC 15 × 103/μL Chloride 100 mEq/L
Hgb 14.2 g/dL CO2 5 mEq/L
Plt 312 × 103 μL Calcium 8.3 mg/dL
Sodium 136 mEq/L UA:
Potassium 2 mEq/L pH 5.5
Chloride 118 mEq/L Specific gravity 1.020
CO2 12 mEq/L Blood Negative
BUN 52 mg/dL Ketones trace ketones
Creatinine 1.5 mg/dL Ethanol 12 mg/dL
Phosphorus 3 mg/dL Glucose 122 mg/dL
Plasma osmolality 280 mOsm/kg H2O Lactate 3 mg/dL
Urine: Plasma osmolality 321 mOsm/kg H2O
pH 6 Phosphorous 3.7 mg/dL
EtOH  Negative
Blood work for ethanol, salicylates, and acetominophen is
Toxicity Screen:
negative. What is the most appropriate treatment strategy
Marijuana +
for this patient?
Benzodiazepine +  
Arterial Blood Gas: A. Aggressive resuscitation with a buffered crystalloid
pH 7.29 solution
Pco2  26 mmHg B. IV fomepizole and emergent hemodialysis
C. Intravenous bicarbonate solution to induce urinary
What is the most likely cause of the patient’s clinical alkalinization
condition? D. IV fomepizole alone

A. Methanol ingestion 3. A 64-year-old man is hospitalized with confusion, nausea,


B. Toluene inhalation and dizziness.
C. Aspirin overdose
D. Alprazolam overdose PMHx: hypertension, atrial fibrillation, hyperlipidemia,
chronic diarrhea
2. A 37-year-old woman with history of diabetes and alcohol PSHx: superior mesenteric artery embolus 2 years ago,
abuse presents in unresponsive state to the emergency s/p resection of large section of small bowel
department. According to her son, she complained of
blurred vision before losing consciousness. Home medications: rosuvastatin, metoprolol, warfarin,
and enalapril; no over-the-counter drugs or supplements
Home medications: metformin

Go_Ch001_p001-p022.indd 13 9/11/18 5:39 PM


14   CHAPTER 1

Vital signs: BP 108/60 mmHg, RR 19 breaths/min, HR 96 Labs:


beats/min, Temp 99°F BUN 45 mg/dL
AAO × 1, confused to place and time Creatinine 2.1 mg/dL
Mucous membranes moist Sodium 135 mEq/L
Lungs clear to auscultation Potassium 3.8 mEq/L
Heart regular rate and rhythm Chloride 106 mEq/L
Abdomen soft and nontender, normoactive bowel sounds CO2 8.4 mEq/L
No edema Calcium 6.8 mEq/L
Glucose 180 mg/dL
Labs: Lactate 2.5 mmol/L
BUN 14 mg/dL Ethanol undetectable
Sodium 140 mEq/L Plasma osmol 320 mOsm/kg H2O
Potassium 3.8 mEq/L
Chloride 106 mEq/L Which of the following statements about this patient is
CO2 17 mEq/L FALSE?
Glucose 90 mg/dL
A. A normal osmol gap would be expected to be less
Lactate Normal
than 10 mOsm/kg.
Plasma osmol 296 mOsm/kg H2O
B. Fomepizole followed by hemodialysis is appropriate
Arterial blood gas: treatment for this patient.
pH 7.37 C. Urine alkalinization via administration of sodium
Pco2 36 mmHg bicarbonate to increase urinary excretion of the toxin
is sufficient treatment for this patient.
Which of the following is the most likely diagnosis? D. Antifreeze is a common source of the alcohol causing
A. D-lactic acidosis this patient’s poisoning.
B. Ethylene glycol or methanol poisoning
C. Propylene glycol toxicity 5. A 24-year-old woman with depression presents to the
D. Pyroglutamic acidosis emergency department with lethargy, confusion, and
vomiting 18 hours after ingesting 100 unknown pills from
4. A 37-year-old man with a history of depression was found a bottle in her parents’ medicine cabinet.
by his son unresponsive in his garage with a suicide note Vital signs: BP 100/55 mmHg, RR 28 breaths/min, HR
lying nearby. On arrival to hospital, patient demonstrated 122 beats/min, Temp 100°F
rapid and shallow Kussmaul breathing with a Glasgow Lungs clear to auscultation
coma score of 4. Tachycardic but regular
Vital signs: BP 110/85 mmHg, RR 26 breaths/min, HR Abdomen soft
100 beats/min, Temp 98.7°F No edema in lower and upper extremities
Pupils reactive to light
Lungs clear to auscultation Labs:
Heart tachycardic and without murmurs pH 7.56
Abdomen is soft and nontender Pco2 22 mmHg
No edema Sodium 144 mEq/L
Potassium 3.2 mEq/L
Arterial blood gas: Chloride 100 mEq/L
pH 6.79 HCO3− 19 mEq/L
Pco2 37 mmHg Creatinine 1.4 mg/dL
Po2 115 mmHg Albumin 5 mg/dL
LFTs within normal limit (WNL)
Drug toxicity screen and testing of salicylic acid and acet-
aminophen were negative.

Go_Ch001_p001-p022.indd 14 9/11/18 5:39 PM


Acid–Base Disorders   15

What is the most accurate characterization of the patient’s


acid–base abnormality?
A. Respiratory alkalosis with concomitant metabolic
alkalosis
B. Respiratory alkalosis with concomitant anion gap
acidosis
C. Metabolic alkalosis with combined metabolic anion
gap acidosis
D. Respiratory alkalosis with anion gap acidosis and
metabolic alkalosis

6. An 84-year-old woman who resides at a nursing home is


sent to the emergency department with 2 to 3 days of
abdominal pain, nausea, vomiting, and low-grade fever.
The patient is severely demented and can provide no his-
tory. Abdominal x-ray (Fig. 1-8) is seen below.
Home medications: diphenhydramine 25 mg, calcium
carbonate 600 mg bid, amlodipine 5 mg, Colace 100 mg, FIGURE 1-8 Abdominal x-ray on admission.
and a multivitamin
Vital signs: BP 105/80 mmHg, RR 18 breaths/min, HR 100
beats/min, O2 sat 99% room air 7. A 31-year-old woman with chronic pain syndrome is
Cachexic elderly woman hospitalized with dyspnea and general failure to thrive.
Clear to auscultation
Vital signs: BP 105/65 mmHg, RR 14 breaths/min, HR 110
Regular rate and rhythm, no murmurs
beats/min, Temp 98°F, Weight 42 kilograms (kgs)
Abdomen is distended, diffuse tenderness to palpation,
Appears anxious and uncomfortable
hypoactive bowel sounds
Temporal muscle wasting
Awake and alert × 1, no focal neurological deficits
Lungs clear to auscultation
Heart tachycardic, but regular rate and rhythm, no
Labs:
murmurs
Sodium 136 mEq/L
Abdomen soft, nontender
Potassium 3 mEq/L
Chloride 90 mEq/L
Labs:
CO2 36 mEq/L
Sodium 130 mEq/L
BUN 56 mg/dL
Potassium 2.0 mEq/L
Glucose 116 mg/dL
Chloride 80 mEq/L
Calcium 10.6 mg/dL
CO2 46 mEq/L
Creatinine 2 mg/dL
BUN 56 mg/dL
Albumin 4.7 mg/dL
Calcium 8.5 mg/dL
Urine chloride 10 mEq/L
Creatinine 1.6 mg/dL
pH 7.47
Albumin 1.9 g/dL
Pco2 51 mmHg
Urine:
What is the mechanism of her acid–base abnormality? Chloride 46 mEq/L
A. Upper GI losses of hydrogen and chloride Potassium 57 mEq/L
B. Diuretic-induced metabolic alkalosis Sodium 46 mEq/L
C. Milk-alkali syndrome
D. Dilutional acidosis What is the likely cause of this patient’s metabolic
alkalosis?
A. Diuretic use
B. Laxative use
C. Intractable vomiting
D. Gitelman syndrome

Go_Ch001_p001-p022.indd 15 9/11/18 5:39 PM


16   CHAPTER 1

8. A 32-year-old man with myotonic dystrophy presents for Which of the following most accurately describes this
follow-up from a recent hospitalization for the treatment patient’s current acid–base status?
of pneumonia. He reports worsening dyspnea over the last
A. Primary respiratory acidosis with appropriate meta-
6 months.
bolic compensation
Vital signs: BP 129/77 mmHg, RR 18 breaths/min, HR 90 B. Primary metabolic alkalosis with appropriate respira-
beats/min, Temp 99 °F tory compensation
Appears anxious and uncomfortable C. Mixed metabolic alkalosis and respiratory alkalosis
Mild crackles heard over the right lower lung field D. Mixed metabolic alkalosis and respiratory acidosis
Regular rate and rhythm, no murmurs
10. A 40-year-old woman with past medical history of diabe-
Arterial Blood Gas: tes and acute lymphoblastic leukemia is admitted with
pH 7.36 respiratory failure due to pneumonia. Labs on admission
Pco2 57 mmHg reveal relatively normal renal function but neutropenia
Po2 85 mmHg (room air) and anemia. Her ICU stay is significant for persistent
HCO3− 31 mEq/L fevers despite appropriate broad-spectrum antibiotic ther-
apy. By hospital day 4, she is started on amphotericin B to
Chest radiograph shows hypoinflation and an improving cover for invasive aspergillosis. On day 8, the following
infiltrate in the upper portion of the right lower lobe. labs were obtained:
Which of the following most accurately describes the
acid–base status and A-a gradient expected in this patient? Labs:
Sodium 144 mEq/L
A. Chronic respiratory acidosis, appropriate metabolic
Potassium 2.6 mEq/L
compensation, widened A-a gradient
Chloride 125 mEq/L
B. Chronic respiratory acidosis, appropriate metabolic
CO2 10 mEq/L
compensation, normal A-a gradient
BUN 35 mg/dL
C. Acute respiratory acidosis, metabolic acidosis, wid-
Calcium 8.2 mg/dL
ened A-a gradient
Creatinine 1.6 mg/dL
D. Chronic respiratory acidosis, concurrent metabolic
Albumin 3 g/dL
acidosis, normal A-a gradient
Urine:
9. A 67-year-old man with COPD and CHF (normal EF) pres- pH 6
ents to the emergency department with shortness of breath Sodium 35 mEq/L
and intermittent associated cough with white sputum. He Chloride 60 mEq/L
reported chronic lower extremity edema. His exam was Potassium 46 mEq/L
consistent with rhonchi and wheezes as well as 2+ lower
extremity edema. His chest x-ray on admission reveals Which of the following is the most likely cause of this
hyperinflation, but no infiltrates or pulmonary edema. patient’s metabolic findings?
A. Type 1 renal tubular acidosis
His Admission ABG: B. Type 2 renal tubular acidosis
pH 7.36 C. Type 4 renal tubular acidosis
Pco2 55 mmHg D. Diarrhea
HCO3− 31 mEq/L
Po2 55 mmHg 11. A 60-year-old woman with type 2 diabetes mellitus (DM)
and hypertension presents to the emergency department
He was started on intravenous steroids, furosemide, bron- with nausea, vomiting, and abdominal pain for 3 days. She
chodilators via nebulizer, and empiric antibiotics. He initially reports an acute diarrhea illness 1 week prior to admission.
improved but on hospital day 3, he appeared more lethargic
and confused. Repeat lab work and ABG was as follows: Home medications: lisinopril, metformin, metoprolol
Vital signs: BP 95/55 mmHg, RR 30 breaths/min, HR 112
pH 7.41 beats/min, O2 sat 97% room air
Pco2 70 mmHg Lethargic, but arousable
HCO3− 43 mEq/L No jugular venous distension
Sodium 142 mEq/L Lungs clear to auscultation
Potassium 3.5 mEq/L Tachycardic but regular rhythm, no murmurs
Chloride 98 mEq/L Abdomen soft, nontender
Creatinine 1.5 mg/dL No edema

Go_Ch001_p001-p022.indd 16 9/11/18 5:39 PM


Acid–Base Disorders   17

Labs: Upon re-evaluation:


Sodium 124 mEq/L
Vital signs: BP 100/70 mmHg, RR 20 breaths/min, HR 95
Potassium 5.5 mEq/L
beats/min
Chloride 85 mEq/L
Appears anxious
CO2 5 mEq/L
No jugular venous distension
BUN 94 mg/dL
Lungs clear to auscultation
Glucose 232 mg/dL
Tachycardic but regular rhythm, no murmurs
Calcium 7.6 mg/dL
Abdomen distended, tender
Creatinine 9.2 mg/dL
Phosphorus 9 mg/dL
Labs:
Albumin 3 mg/dL
Sodium 132 mEq/L
Lactic acid 10.7 mmol/L
Potassium 2.5 mEq/L
pH 7.15
Chloride 80 mEq/L
Pco2 15 mmHg
CO2 41 mEq/L
BUN 35 mg/dL
A CT of the abdomen and pelvis with contrast revealed
Calcium 6.0 mg/dL
no evidence of mesenteric ischemia, colitis, or other intra-
Creatinine 1.6 mg/dL
abdominal process. Which of the following statements
Albumin 2.3 g/dL
about the treatment of this patient with sodium bicarbon-
ate is FALSE? Arterial Blood Gas:
pH 7.49
A. Sodium bicarbonate therapy can result in hyperna- Pco2 50 mmHg
tremia and fluid overload. Po2 60 mmHg
B. Sodium bicarbonate therapy can result in hypocalce-
mia and worsen myocardial contractility. What is the most likely pathogenesis of the patient’s cur-
C. Sodium bicarbonate therapy can increase generation rent condition?
of carbon dioxide.
D. Sodium bicarbonate can aid in renal excretion of A. Metabolic alkalosis due to vomiting
metformin via ion-trapping in tubules. B. Metabolic alkalosis due to conversion of excess
citrate to bicarbonate via the tricarboxylic acid
12. A 50-year-old man presents to the emergency department (TCA) cycle
with trauma to the abdomen following a motorized vehicle C. Respiratory alkalosis secondary to hyperventilation
accident. due to uncontrolled pain
D. Underlying pulmonary embolism as a complication
Vital signs: BP 80/55 mmHg, RR 30 breaths/min, HR 124 of recent abdominal surgery
beats/min, O2 sat 97% room air
Lethargic, but arousable
No jugular venous distension ANSWERS
Lungs clear to auscultation
Tachycardic but regular rhythm, no murmurs 1. B. Toluene inhalation
Abdomen distended, tender
Toluene is widely used in many industrial solvents, acrylic
Labs: Pending paints, and paint thinners. Toxicity may be a result of envi-
ronmental, accidental, or intentional exposure. It is the most
On abdominal CT, spleen demonstrates branching hyper-
widely abused inhaled volatile drug; it causes a euphoric
densities, consistent with rupture. A splenic subcapsular
effect when sniffed. Toluene is metabolized to benzoic acid
hematoma is also noted. He is transfused 2 units of packed
and then to hippuric acid. These anions are very readily
red blood cells and taken to the operating room where he
excreted into the urine as sodium and potassium salts. Most
undergoes splenectomy with no complications. During
patients with toluene ingestion present with hypovolemia,
his procedure, an additional 3 units of packed red blood
hypokalemia, and a normal anion gap acidosis.37
cells are dispensed.
Methanol ingestion (and frequently ethanol) would cause
Overnight, the patient is given broad-spectrum antibiot-
a high anion gap acidosis (choice A). Aspirin overdose
ics, morphine, norepinephrine, and normal saline. He
causes a mixed respiratory alkalosis with high anion gap
requires another 5 units of packed RBC overnight. The
acidosis (choice C). Benzodiazepine overdose typically
following morning, the patient is found to be anxious and
presents with a respiratory acidosis due to suppression of
has vomited twice.
respiratory drive (choice D).

Go_Ch001_p001-p022.indd 17 9/11/18 5:39 PM


18   CHAPTER 1

2. B. IV fomepizole and emergent hemodialysis 3. A. D-lactic acidosis


Methanol, also known as wood alcohol, is sometimes con- D-lactic acid is produced by fermentation of carbohy-
sumed as a substitute for ethanol, and initial presentation drates by colonic bacteria. The body typically produces
after ingestion resembles ethanol intoxication. Patients L-lactic acid, and our endogenous lactate dehydrogenase
may present with drowsiness, seizures, or vision changes cannot metabolize the D-lactate variant. Increased absorp-
or loss. A report from the American Association of Poison tion of D-lactic acid typically occurs in patients with short
Control Centers stated that 44 out of 979 methanol poi- bowel syndrome; it can also occur in cases of high carbo-
soning victims had major complications.38 hydrate load or decreased colonic motility. D-lactic acido-
Methanol is metabolized by the body first to formalde- sis causes mental status changes and patients present with
hyde by alcohol dehydrogenase and then formic acid encephalopathy.2 This condition can also present with a
(Fig. 1-9). Formate is the unmeasured anion that causes normal anion gap acidosis due to renal excretion of lactate
the elevated anion gap and is toxic to the retina, causing with a cation such as sodium.
the visual changes classically associated with methanol Choices B and C are incorrect, as they should have an
toxicity. elevated osmolal gap (> 10 mOsm/kg H2O). Pyroglutamic
Fomepizole (Antizol) is the antidote for both methanol acidosis (choice D) causes a gap acidosis in the setting of
and ethylene glycol poisoning. It works by competitively chronic acetaminophen use, which this patient denies.
inhibiting alcohol dehydrogenase and thereby preventing
the conversion of methanol to its toxic metabolites. This 4. C. Urine alkalinization via administration of sodium
slower rate of production allows the liver to process the bicarbonate to increase urinary excretion of the toxin is
metabolites at a manageable rate and prevents organ sufficient treatment for this patient.
damage. Ethylene glycol is an alcohol found in antifreeze, de-icing
In severe methanol poisoning, hemodialysis should be solutions, and windshield wiper fluid. Due to its sweet
used to rapidly remove methanol from the body. The taste, it is often ingested by children, alcohol abusers seek-
combination of hemodialysis and medical treatment ing an alternative to alcohol, and by persons attempting
(fomepizole) has been shown to decrease mortality and suicide. Its primary metabolites, glycolic acid and oxalate,
permanent neurological damage in severe cases.14,38,39 are the unmeasured anions responsible for the high anion
Hemodialysis is recommended in any patient with meta- gap metabolic acidosis. In cases where ingestion is sus-
bolic acidosis or manifesting evidence of end organ dam- pected, laboratory testing for ethylene glycol specifically
age (renal failure or blindness). Another option that is not may not be available, so clinicians should use other labora-
listed is that intravenous ethanol is an effective alcohol tory findings to make a diagnosis; this is where measuring
dehydrogenase inhibitor. However, observational studies an osmolal gap is useful (choice A).
suggest that ethanol has a much higher incidence of As in the treatment for methanol ingestion, fomepizole
adverse reactions compared to fomepizole. The main can be given to block the conversation of ethylene glycol
adverse effect associated with ethanol is central nervous to its toxic metabolites (choice B). Sodium bicarbonate
system depression.40 may increase urinary excretion and decrease tissue toxic-
Choice A is incorrect, as simple volume resuscitation, ity; however, it should not be used without fomepizole or
although indicated, is insufficient treatment for toxic alco- hemodialysis (in this patient with end organ damage). It
hol ingestion. Intravenous bicarbonate (choice C) is useful should also be noted that sodium bicarbonate administra-
for short-term treatment of acidosis, but in the setting of tion may paradoxically further nephrotoxicity by raising
such severe renal failure, it will not treat methanol inges- urine pH and increasing the precipitation of oxalate crys-
tion adequately. tals in the kidney (choice C).41-43

Methanol Formaldehyde Formic acid

O O
H OH
Aldehyde dehydrogenase
C C C
Alcohol dehydrogenase
H OH H OH H OH

Fomepizole
FIGURE 1-9 Conversion of methanol to formic acid.

Go_Ch001_p001-p022.indd 18 9/11/18 5:39 PM


Acid–Base Disorders   19

5. D. Respiratory alkalosis with anion gap acidosis and suspect surreptitious diuretic use in this nursing home
metabolic alkalosis patient. Although the patient has a slightly high calcium,
Aspirin overdose classically presents with hyperventila- which might suggest milk-alkali syndrome (choice C), her
tion, gastric irritation, and tinnitus. Supratherapeutic calcium is not high enough to cause AKI, and in fact, her
doses of salicylate directly stimulate the respiratory center high calcium level is likely due to hemoconcentration
in the medulla. Therefore, aspirin toxicity produces a pri- (high albumin level). The patient does not have a dilu-
mary respiratory alkalosis along with an anion gap acido- tional acidosis; in fact, she has a contraction alkalosis
sis. As a result of this mixed picture, blood pH may be (choice D).
within normal limits in the setting of increased anion gap.
7. A. Diuretic use
The acidosis produced by aspirin overdose is multifacto-
rial. In addition to being an endogenous acid itself, aspirin For patients presenting with nonspecific symptoms, a care-
causes uncoupling of oxidative phosphorylation and inhi- ful history and physical examination (with key portions,
bition of the Krebs cycle. This inhibition results in an accu- including vital signs, body mass index (BMI), and parotid
mulation of organic acids and an increased production of gland swelling) are important to assess. In this case, this
lactic acid. Aspirin can also impair renal function, which patient’s BMI and physical exam are consistent with an
results in further accumulations of organic acids, such as eating disorder with associated dehydration/electrolyte
phosphoric and sulfuric acids. abnormalities. Her urine chemistries, however, suggest
that her alkalosis is due to diuretic abuse rather than vom-
There is no antidote for aspirin, so the goal of therapy is to iting. Her urine chloride is greater than 40, which suggests
limit absorption and enhance elimination. Patients are a chloride-resistant metabolic alkalosis. In this patient,
treated with gastric lavage, activated charcoal, and sup- who likely suffers from an eating disorder, diuretic use has
portive measures, such as hydration and correct acid–base to be suspected. Diuretics (both thiazide and loop diuret-
disturbances. The airway should be stabilized and mechan- ics) block the kidneys’ ability to appropriately resorb
ical ventilations provided, if required. Hemodialysis may sodium chloride, which results in an increased urine chlo-
be indicated in severe cases.44-46 ride concentration. In addition, the increased delivery of
Analysis of this patient’s acid–base status is complex. Anal- sodium to the distal tubule leads to sodium reabsorption
ysis of the arterial blood gas reveals a respiratory alkalosis at the convoluted tubule (activated by aldosterone) and
with acidosis slightly greater than expected; analysis of the excretion of potassium in exchange, which partially
serum chemistry shows an anion gap of 25, which suggests explains this patient’s profound hypokalemia.
an elevated anion gap acidosis. Calculation of the delta Vomiting often presents with a concurrent hypovolemia and
anion gap/delta bicarbonate ratio suggests that the fall in stimulates renin and aldosterone activity. As a result, the
bicarbonate was much less than the increase in the anion kidneys actively resorb Na, HCO3−, and Cl, thus reducing
gap (ratio > 3), which suggests a concomitant metabolic the amount of urine Cl to less than 25 mEq/L (choice C).
alkalosis (probably from vomiting). Answer choices A, B, In contrast, laxative use depends on the mechanism of the
and C are incorrect. drug of choice, but usually causes loss of bicarbonate in the
diarrhea, and causes a non-gap metabolic acidosis with
6. A. Upper GI losses of hydrogen and chloride hypokalemia (choice B). Although Gitelman syndrome
This patient’s x-ray showed peristaltic ileus. Due to her would present with a chloride-resistant metabolic alkalosis,
nausea and vomiting, it is likely that she has metabolic it is usually diagnosed in children and would not explain
alkalosis due to gastric losses of H+ and Cl−. Her volume- the physical exam findings in this patient (choice D).
depleted state causes high aldosterone levels and activation
of the type A intercalated cell, which stimulates hydrogen 8. B. Chronic respiratory acidosis, appropriate metabolic
secretion (see Fig. 1-4). The high level of aldosterone in compensation, normal A-a gradient
combination with gastric losses leads to a hypokalemic Mechanisms that affect A-a gradient include ventilation/
metabolic alkalosis. perfusion (V/Q) mismatch, right-to-left shunting, diffu-
In evaluation of metabolic alkalosis, urine chloride is help- sion limitation, hypoventilation (drugs, obesity, etc), and
ful. Urine chloride less than 25 mEq/L can suggest gastro- reduced inspired oxygen tension. In this example, muscu-
intestinal loss, contraction alkalosis, and late diuretic use. lar weakness results in a pure hypoventilation syndrome,
Urine chloride more than 40 mEq/L can suggest primary which results in a chronic respiratory acidosis. There
hyperaldosteronism, hypokalemia, Gitelman syndrome, would be metabolic compensation, as this patient does not
and Bartter syndrome. Diuretic-induced metabolic alka- have renal failure or metabolic insufficiency. There is a
losis (choice B) is incorrect due to the low urine chloride. simultaneous acute respiratory acidosis as a result of infec-
Diuretics could cause a low urine chloride if the diuretic tion and pulmonary effusions as described by radiography
has not been given in 24 hours, but there is no reason to and physical exam. The A-a gradient would be normal.

Go_Ch001_p001-p022.indd 19 9/11/18 5:39 PM


Another random document with
no related content on Scribd:
The Project Gutenberg eBook of Metsolassa
This ebook is for the use of anyone anywhere in the United States
and most other parts of the world at no cost and with almost no
restrictions whatsoever. You may copy it, give it away or re-use it
under the terms of the Project Gutenberg License included with this
ebook or online at www.gutenberg.org. If you are not located in the
United States, you will have to check the laws of the country where
you are located before using this eBook.

Title: Metsolassa

Author: Oskari Hynninen

Release date: December 15, 2023 [eBook #72424]

Language: Finnish

Original publication: Helsinki: Yrjö Weilin, 1903

Credits: Juhani Kärkkäinen and Tapio Riikonen

*** START OF THE PROJECT GUTENBERG EBOOK


METSOLASSA ***
METSOLASSA

Kirj.

Oskari Hynninen

Helsingissä, Yrjö Weilin, 1903.

SISÄLLYS:

Metson soitimella.
Nevalla.
Heija-Pekko.
Eläimiemme talvipuvuista.
Koiralleni.
Alleja ampumassa.
Mateita pyytämässä.
Miten kesäpäiväni viettäisin.
Syysmuistoja.
Talvinen metsä.
Merilintuja.
Luvattomalla ajalla.
Hylkeenhuudossa.
Metsolassa.
Vappuna.
METSON SOITIMELLA.

On yhä olemassa joukko vanhempia metsämiehiä, jotka haaveksien


muistelevat menneitä aikoja, jolloin ei mikään metsästys-asetus
pakoittanut lintumiestä puoleksi vuodeksi joutilaana olemaan.
Tunnen vanhoja soitimella kävijöitä, jotka loistavin silmin kertovat,
miten »hiovan» metson kevätaamun sarastaessa männystä
pudottivat — — ‒. Ja silloin muistelmiensa innostamana kertoja
oikein runolliseksi rupeaa, kun kuusikon rakeisesta sulalumesta ja
korven hienosta aamuhämärästä, kevätpuron salaisesta solinasta
tahi lahorastaan haastelusta juttuamaan pääsee.

Nyt on toisin! Emme saa enään murha-ase olalla keväisiä metsiä


samoella — — ‒ eikä kanalintumme hääiloa häiritä. Nyt saattavat ne
soidintaan lain turvissa viettää — ja hyvä niin onkin!

Mutta varmaan tahtoisi joku nuorempikin luonnonystävä ja


metsämies tutustua Tapion kukon häämenoihin, omin korvin kuulla,
miten metso hämärässä korvessa juhlavirttään virittelee tahi suuren
salon hiljaisuudessa lemmenlauluaan laskettelee!

Ei tuo huvi nykyoloissakaan ole mahdotonta!


Sillä ei metso laula ainoastaan keväällä — se pitää soidinta
silloinkin, kun luonnossa ensimmäiset syysenteet huomaamme. Se
ei liene aivan kaikille tuttua.

Kuhertaahan urosteerikin syksyllä, kun hallayöt alkavat


lehtimetsän vihreyden keltaiseen vaihtaa, ja harmajat aamu-usvat
pehmeinä laaksoissa lepäävät. Lasketteleehan lehtokurppakin
lämpimän syysillan hämärässä — niinkuin keväällä ainakin — »kurp-
kurppiist» lepikosta toiseen nokka ojona lekutellessaan. Entäs
metsäkana! Kun kirkkaankylmä syysilta pimeytensä yli rämeiden
kellastuneen saraheinän levittää, silloin se pajukossa kevätääniään
päästelee, »kopeekoita lukee» ja — luoksesi tulee, jos naaraan
naukuvaa ääntä matkia osaat.

Ei metsokaan tässä suhteessa poikkeusta tee!

Kun on ruis leikattu ja syyskylvöt tehty, silloin kokenut


metsovanhus huomaa kesän ohitse olevan. Kylmemmiksi käyvät
aamut, auringon noustessa kimaltelee kuura maassa ja
kevääntapaista raittiutta on luonto täynnä.

Silloin ukkometson rinnassa keväiset muistot ja taistelu-into


uudelleen elpyvät ja vastustamattomasti aamuhämärässä heikko
»kaputus» ilmoille pääsee. Puuhun erakko kuhahtaa, ja kohta
keväälliset riemulaulut, metsonsoitimen eri sävelet kuulumaan
rupeavat.

Nyt saatat metson soitimelta ampua, jos tehtäväsi tunnet ja —


maltillisena pysyt.

Keväästä vanhus laulaa — joka kului, kuudesta kirjavasta


kullastaan, koppeloistaan, joitten tähden niin kovasti taistella täytyi,
naaraistaan, joiden kanssa kanervikossa iloja piti — — — ja silloin
vanhus miltei silmänsä ummistaa, ei kuule eikä paljon näekään, ja
nyt sitä lähelle pääset.

Eivät arat naaraslinnut nyt laulavaa urosta kotkotuksillaan varoita,


vaan marjamäillä ja metsäpurojen varsilla poikineen huolenpäiviä
viettävät. Ei nyt hiiviskelevän metsästäjän huomannut koppelo kohti
»hiovaa» metsoa lennä ja siivellään sitä sivahuta, että sekin ajoissa
vaaran huomaisi ja lentoon lähtisi — niinkuin niin monasti keväällä
tapahtuu. Yksinään vanhus kummallista ylistysvirttään syksyllä
soittelee ja sentähden on sen menoja siihen aikaan helpompi
tarkastaa.

Tee koetteeksi näin!

Kun ilta elokuun lopulla tahi syyskuun alussa on tyyni ja usvainen,


lähdet jonkun kokeneen metsämiehen kanssa, sellaisen seurassa,
joka Tapion kukon tavat ja hakolinnun asuinpaikat tarkoin tuntee,
rämeniityn laidassa olevaan latoon yöpymään. Kahisevien
kuloheinien peitossa nukutte muutaman tunnin ainoastaan toinen
silmä ummessa — sillä pimeässä, jo ennenkuin päivä sarastaa, on
liikkeelle lähdettävä.

Mustana hämöittää metsä rämeen laidassa ja syksyinen


äänettömyys vallitsee. Ei laula kyntörastas nyt kuten keväällä, ei
visertele punarinta satakieli kuusikon suruvoittoisia säveliä.
Ainoastaan pieksun askel pehmeässä: rahkasammalessa kuiskaten
kahisee, joku oksa risahtaa… Kuuloa aina äärimmäisyyteen
jännittäen kuljette askel askeleelta eteenpäin. Väsynyt korva yhä
pimeyteen kuulostelee, josko toivottua metson »kaputusta» jostain
kuuluisi — — —.
Jopa vihdoinkin!

Omituinen ääni — aivan kuin lappeettain vasten pyssynpiippuja


olevaa puukonterää varovasti tärähytteleisit tahi kahta kovaa
palikkaa vastatusten naputteleisit — — — kep-pep, kep-pep, jos
kirjaimilla tahtoisit ääntä jäljitellä.

Samalla huomaat päivänkin heikosti koittavan. Tummanharmajaksi


käy ympäristö. Jo alat eroittaa kuusikon lehtimetsästä. Kuuluu ison
linnun siiven läimäys. Unisesti ensimmäinen rastas räkättää.

Toverisi kintereillä hiivit mieli jännityksessä, samoihin jälkiin


tallaten. Pysähdyt silloin, kuin hänkin, astut eteenpäin, kun hän astuu
— — —. Jo alat eroittaa eri sävelet metson laulussa ja nyt alkaa
vaikein tehtävä, sillä nyt on varovasti lähestyminen. Ensin kuulet
tuon äskeisen kep-pep, kep-pep ja sitten sama uudistettuna, mutta
hyvin nopeassa tahdissa — se on »käkkärä», johon yhtyy »litkaus».
Yhä vielä seisot paikallasi, totisesti maahan katsoen, tämän osan
loppua odotellen, sillä heti seuraa »hiominen» ja silloin otat kaksi,
kolme pitkää askelta eteenpäin. Korvassasi kuuluu hiominen kuin
"hiisi-hiisi-hiisi", jolloin vanhus hurmoksiin menneenä tämän
maailman miltei kokonaan unhoittaa. Mutta varo, että ennen
hiomisen loppua taas seisot kuin pölhö jonkun sopivan puun
suojassa. Sillä muutoin lintu lähtee heti. Älä myöskään lähde
liikkeelle, ennenkuin hiominen on alkanut, sillä näin syksyllä vanha
metso useinkin päättää laulunsa »käkkärällä» ja pysähtyy
kuuntelemaan… Ja tätä merkillistä kolmiloikkausta kestää kauvan,
mielestäsi sietämättömän kauvan — — —. Kaikesta päättäen pitäisi
sinun jo olla hyvällä ampumamatkalla, vaan et näe mitään.

Kuulet sydämesi tykytyksen, kuulet, miten vaatteet hengittäessäsi


ruumistasi kihnuttavat, tunnet, miten hikipisara toisensa perästä
selkää myöten alas juosten ihoa kutkuttaa — — —.

Vaan pysy levollisena!

Valkenemistaan päivä valkenee ja metsä valveutuu. Jo vihdoin


huomaat hämäräisessä männyssä vanhuksen, kaula paksuna kuin
turkin hiha, pyrstö levällään kuin rukin pyörä, siivet oksia viistäen ja
käyrä nokka hiomisen kestäessä taivasta kohti ojennettuna.

Kannattaa sitä katsella… ja kun olet kylliksesi sitä ihaillut, ojennat


— linnun juuri hioessa — pyssysi ja pudotat vanhuksen, joka kovasti
rymisten ja oksia katkoen maahan muksahtaa.

Olet tullut tuntemaan salomaan salaisuuksista yhden, astunut


korven kummallista elämää askeleen lähemmäs.
NEVALLA

Oletko seisonut pohjoissuomalaisen nevan laidassa antaen silmäsi


liidellä yli peninkulmia käsittävän autionnäköisen lakeuden, yli
silmänkantamattomiin ulottuvan, vaivaiskoivua, rahkasammalta ja
kituvia mäntyjä kasvavan aavikon. Oletko nähnyt noita toivottomia,
ikäänkuin nälkääntyneitä rämeitä alkuperäisessä tilassaan, ilman
kirveen tai kuokan jälkiä, yökylmien kotipaikkoina, mistä härmäinen
halla viljamaille hiipii ja kylmään kouristukseensa maamiehen vaivat
sulkee?

Puoleensa ne ainakin minua vetävät, miellyttäviä ne ovat kaikessa


kolkkoudessaan ja autiossa köyhyydessään! Sillä vasta siellä
tunnen, miten luonto on suuri ja ihminen — vähäinen.

Ei ole nevakaan vaihtelua vailla, ei puutu rämeeltäkään suloutta!


Erilaiset ne ovat kukoistuksensa hetkinä, toisenlaiset lakastumisensa
aikoina. Joka vuodenaikana — jopa talven kuolinvaippaankin
puettuina ne minulle nähtäviä tarjoavat.

Kun kevätlumi sulavetenä solisten kohti nevaa rientää, silloin se


tulvan vallassa järvenä lainehtii. Silloin ei silmäni sen vastaista
rantaa eroita. Kehyksenä on nääntynyt, naavaturkkinen kuusikko,
joka kauvempana yhä matalammaksi käy, näyttää veteen painuvan,
avaten näköaloja aina loppumattomiin. Saarina kuihtuvaa mäntyä
kasvavat kaarrot ruskeasta vedestä kohoavat. Ja kun laskeva
aurinko tulvaveden kultiin pukee ja kevätilma on tyyni ja raitis, silloin
kaarroilta kuulen luonnon ylösnousemisjuhlaa vietettävän — kuulen
ääniä kummallisia mitä miellyttävimmässä vaihtelussa.

Kuusikon laidassa laulurastas sanarikkaita säveliään laskettelee


tynnyrilinnun tilt-talttia takoessa. Siihen yhtyvät urostavin rinkutus
sekä viklain vihellykset. Ulompana kurjen kaikuva torvensoitto
ympäri rämettä kajahtelee, ja ruskopilvistä kuuluu lepoa hakevien
metsähanhien kaakatus. Tyynessä vedenkalvossa uiskentelee
vesipääskynen keveänä kuin höyhen, jota tuuli sinne tänne aalloilla
ajelee. Ja kun pohjolan kevätyön hieno hämärä hiljalleen rämeelle
laskee, silloin pikku varpuspöllö kolmiäänistä nuottiaan vinguttaa,
ikäänkuin nevalla soudettaisiin ja airojen hangat naukuisivat.

Mutta jonkun viikon kuluttua järvi katoaa, sorsat häviävät ja


vesipääskynen lähtee pohjoisemmille pesimäpaikoilleen.

Näeppä silloin samat seudut alkukesän kirkkaudessa, kun neva


pukeutuu ensimmäiseen vienonvaatimattomaan kasvullisuuteensa.
Järvestä on tullut tuhatmättäinen tasanko, joka punervankainona,
vaikka hieman suruvoittoisena vastaasi hymyilee ja kasvilajeista se
on rikkaampi, kuin luulisikaan.

Ensimmäisenä nousee sammalpeitteestä tupellinen niittyvilla —


mustapää — ja kohta alkavat monet pajulajit kukkia. Vaivaiskoivu
tekee lehteä, suokukka siroja punakellojaan soittelee, kun linnut
häitä hankkivat. Jopa kaartojen kanervat, variksenmarjat,
peuranjäkälät ja itse valkosammalkin näyttävät elpyvän. Ja lukuisat
sarat, juurto-, hivus-, kasti-, liejusarat — kukapa niitä kaikkia jaksaa
luetella — vihvilät, piirtoheinät rientävät kohti korkeutta.

On nyt siitepölyä ilmassa ja rakkautta joka mättäällä, riemua ja


huolta, taistelua ja kilpailua joka pajukossa, ääntä
vaivaiskoivuviidakossa, sillä kesä loihtii elämää kaikkialle.

Nevan lintu, nevanvärinen tunturikurmitsa surumielisiä, valittavia


ääniään päästelee, nevan yksinäisyyttä ylistellen. Iloisesti valkea
vikla hyklyä huutaa ja pikkukuovi pitkään viheltää. Mättäältä kohoten
niittykirvinen vaatimatonta säveltään sirittää, pajukossa urosriekko
pesivälle naaraalleen pontevasti puhelee. Mutta kuivettuneen
männyn latvassa istuu kaikkien vihollinen, saaliinhimoinen
muuttohaukka, terävällä silmällään tarkastellen ääretöntä
metsästysaluettaan, joka kohta muurainten kukista valkeankirjavana
loistaa. Ja mättäiden väliin, missä harmaa kyy kuin luikerteleva
vesisuoni matelee ja sisiliskot mennä viilettävät, ilmestyy
vaatimattomia pesiä ja ruskeankirjavia munia, joiden kuoressa
jäkäläin ja turvemullan väri on vallitsevana.

Nyt hyönteisetkin lyhyttä ilonaikaa viettävät. Lukemattomina


laumoina sääsket hienon musiikin soidessa häätanssiaan survovat ja
yninä kasvaa mahtavaksi kööriksi miljoonien väristimien
väräjöidessä.

Mutta kun suopursut tuoksumaan ja muurainmättäät kellertämään


rupeavat, kun kaartojen puolukat punottavat ja juolukanmarjat
mustuvat, silloin alkaa kesän komea kirjavuus kadota. Ei ole enään
syksy kaukana. Kohta alkavat pesineet linnut poikinensa hävitä, jo
alkaa nevan elämä alakuloisemmaksi käydä. Kukkineet sarat
kellastuvat, ja kohta rakentaa ensimmäinen yöhalla kimmeltäviä
kuurakiteitä jokaiseen korteen — —.
Jos näet nevan nousevan syysauringon raikkaassa valokylvyssä,
voi yksinäisyyden ja kaihon tunne mahdollisesti hetkeksi mielestäsi
haihtua. Reippaasti raikuvat lähtöä tekevien kurkien huudot ja
nevanlaidassa kuhertaa, puhaltaa mustansinertävä, lyyrypyrstöinen
urosteeri. Mutta kun taivas on tasaisen harmajana, kun se matalana
miltei metsänlatvoihin yhtyy ja viikkokausia kestänyt vihmasade
supistaa näköpiirin, silloin hallanpurema neva on sanomattoman
surullinen, köyhä, autio.

Ruskean turveveden ympäröimänä kohoaa mätäs toisensa takana


aina loppumattomiin. Alakuloisesti nuokkuu kellastunut sara
kolkossa pohjatuulessa, joka kohisten vaivaiskoivikossa syksyn
kannelta soittaa ja siltä lehdet riistää. Ruskeata ja harmajaa — kas
siinä nevan värit tähän aikaan! Ainoastaan mättään kyljessä
punottava karpalo tahi lätäkköön hetkeksi levähtämään laskeutunut
tavi suovat silmälle vaihtelua.

Kun itätuulet mustia lumipilviä yli nevan kiidättävät ja ensimmäiset


hiutaleet valkeina, keveinä, äänettöminä putoilevat, silloin luonto
nevalle kuolinvaippaa kutoo. Ja kohta lepäävätkin sarat ja sammalet,
kanervat ja vaivaiskoivut alla kylmän valkopeitteen.

Mutta ei nevalta puutu elämää silloinkaan. Jäniksen jälkiä kulkee


viidakosta toiseen, metsäkanoja juoksentelee pitkin lumenpintaa,
kettu käy yöllisillä metsästysmatkoillaan nevaakin tervehtimässä ja
keveillä, äänettömillä siivillään leijailee valkea tunturipöllö pitkin
ääretöntä lakeutta, tavotellen pientä päästäistä tahi valkoista
metsäkanaa.

Mutta surullinen on talvipukuinen neva, toivottomalta se näyttää


puolihämärän talvipäivän lumivalossa tahi kelmeän revontulen
välkynnässä. Toivottomia tunteita se herättää — tahtoisi miltei
heittäytyä tuohon pehmeään peitteeseen ja levätä siinä, odottaen
kevättä, joka tuntuu olevan niin kaukana, kaukana. Nukkua
äänetönnä ja herätä lämpimiin kevättuuliin, kurjen mahtavaan
huutoon, viklojen liverryksiin — — — — —
HEIJA-PEKKO

Taru kertoo, että kun Väinämöinen kantelonsa kieliä helähytteli,


vaikutti soitto sekä ihmisiin että eläimiin, jopa metsän pedotkin sekä
ilman linnut kokoontuivat iloa ikirunojen kuulemaan.

Ei ole meillä enään soittajaa sellaista, joka saattaisi sekä eläinten


että ihmisten mielet lumota. Emme enää osaa luonnon ääniä tulkita
eikä luonnon äänillä puhua, niinkuin Väinämöinen kanteleellaan
saattoi. Siihen olemme liian jokapäiväisiä tahi liian — uudenaikaisia,
eläen vieraina luonnolle, tuntematta mitä ympärillämme tapahtuu,
syventyneinä kukin omiin tehtäviimme. Harvallapa lienee enää
korvaa, jolla saattaisi luonnon suurta sinfoniaa käsittää tahi sen
hämäristä, salaperäisistä soinnuista täysin nauttia.

Tunnen kumminkin yhden, joka on perin tutustunut keväisen


metsän yhteissoittoon, perin perehtynyt luonnon laulajien
puhelutapaan ja lintujen keskusteluihin. Hän ei ole mikään tiedemies,
ei hän ole käynyt mitään musiikkiopistoa, sen voi jo päättää hänen
vaatimattomasta nimestään — Heija-Pekoksihan kansa häntä
nimittää.
Mutta, niinkuin hän itse puoleksi kehuen sanoo: »Jumalan
antamana lahjana» hän on kykyään kehittänyt, öitä päiviä korvessa
viettänyt oppiakseen ymmärtämään ja puhumaan lintujen kieltä — ja
hän oppi!

Kerronpa tässä, miten Pekko oppivuosia metsissä vietti ja


mestaruuden alallaan saavutti.

Karjalassa Pekko syntyi, paikkakunnalla, missä laulu elää vielä


tänäänkin kansan huulilla. Ruotulapsena hän kasvoi ja jo poikana
pahaisena tehtiin hänestä paimen. Seitsemästä kopeekasta taloa
kohti hän keväästä syksyyn saakka piti huolta suuren kylän
lehmikarjasta, suojellen kutakin suohon sotkeutumasta, kontion
kynsiin tahi suden suuhun joutumasta.

Kun aamulla karjankellojen kalkattaessa kylän kujansuusta


lähdettiin, silloin pääskyt Pekolle jäähyväisiään visersivät ja kiurut
raittiista korkeudesta aamutervehdyksiään livertelivät. Kun
takalistoon tultiin, siellä käki väsymättä parhaimpia
kevättervehdyksiään lasketteli. Ja ahoilla sekä rinteillä päivää
viettäessä, soita ja saloja samoillessa hän kuuli aina uusia ääniä,
milloin tuttuja, milloin outoja, milloin niin kaameasti pöyristyttäviäkin,
että miltei saivat veret suonissa tyrehtymään, itkun kurkkuun
nousemaan.

Korpisuossa hän kuuli, miten soitimen iloja ja ponnistuksia


muisteleva metso viimeisiä kaputuksiaan kalahutteli, kaskimailla,
miten urosteiri kevään loppuessa muniville naaraille häätanssin
nuottia pulitteli, ja rämeen reunassa, vaivaiskoivikossa metsäkana
miehen ääntä matkien uhkaili: »poika vätkylä — poika vätkylä»,
»parrtaa pois — parrtaa pois», »koko pää — koko pää», jonka
jälkeen se nauraa kakatteli kuin mielipuoli, että oli aivan kuoliaaksi
peljästyttää — — —! Mutta rämeen päällä sinertävässä korkeudessa
taivaanvuohi mäkätti kuin paholaisen pässi tahi velhottu vuohi —
koska aina näkymätönnä pysyi.

Hän kuunteli noita kummallisia ääniä ja koetti niitä ymmärtää — —


‒. Ja vähitellen Pekko oppikin niitä käsittämään. Tutuiksi tulivat
hänelle metsän humisevat salaisuudet, ystäviksi sen arat eläjät
toinen toisensa jälkeen muuttuivat.

Äänestä hän kuuli, milloin räkättirastas oli hädässä, kun varis


tahtoi pojat viedä, riensi apuun ja harmaatakkinen rosvo pakeni. Hän
tiesi tiaisen piipityksestä, milloin varpushaukka oli läheisyydessä, ja
huomasi kuinka muutkin pikkulinnut ymmärsivät ottaa varoituksesta
vaarin. Ja leppälinnun hätähuudosta hän käsitti, milloin käkönen
tahtoi varkain pistää pahanonnen munan vieraaseen kotiin.

Näin Pekko vähitellen pääsi lintujen kielistä perille, mutta — ei


osannut puhua mitään vastaukseksi. Hän koetti kurkullaan, vaan
äänivarat olivat riittämättömät. Hän koetti viheltää, mutta huulet eivät
teroittuneet. Koettipa karaista vihellystään tuohipääpuukkonsa
kärkeä vastaan — ääni parani kyllä vähin, mutta peipposen viserrys
oli kumminkin sointuvampi.

Kerran, kun koivu taas seisoi vastapuhjenneessa keväisessä


lehvässään, kiskoi hän sen vyötä valkeaista paimenvirsuaan
paikatakseen. Tuohi juoksi mielellään ja vihreä aluskuori jäi runkoon.
Jäipä sen päälle vielä hienon hieno kalvo, läpikuultava ja pehmeä
kuin kananmunan kelmu. Sen hän irroitti, katseli sitä hellästi
tarkastellen, miltei ystävyydellä, sillä hän rakasti koivujaan ja sääliksi
kävi, kun toveriltaan, mielipuultaan riisti sen valkean kauneuden…
»Jospa kalvon ohut reuna paremmin kuin puukonterä vihellystäni
karkasisi» — — ‒! Hän koetti — ja siitäpä tuntuikin! Kuinka olikaan,
joutui pehmeä kalvo huulien väliin ja kun hän huulensa avasi, kuului
maiskahtava ääni — aivan kuin mustasukkainen satakieli
hämyisessä lepikossa maiskuttelee. Kokeillen sälyillään, niitä
huuliensa välissä milloin tiukentaen milloin höllentäen, syntyi aina
uusia ääniä — milloin minkäkin linnun liverryksiä… »Kas tässä pilli,
jolla voin vastata lintujen puheluun!»

Ja päivämääriä harjoitettiin yhä paremmalla menestyksellä…

Oltiin taas metsässä laulavassa, kuusikossa kukkuvassa.


Käenpiika kutsui kultaansa aurinkoisessa lehdossa… Pekko koetti
vaatimatonta konettaan, ikävöivän linnun ääntä matkien — ja katso!
Heti oli käenpiika hänen luonaan, luottavaisesti tirkistellen Pekkoa
alimmalta koivunoksalta. Läheisessä kuusikossa kyntörastas
puoliääneen ja niinkuin hajamielisenä harjoitteli, ollakseen täysissä
äänivaroissa illan tullen. Pekko puhui kyntörastaalle: »tuu liki, tuu
liki», ja saikin heti vilkkaan vastauksen, kunnes lintu tuli luo,
tutustuakseen uuteen toveriin. Mutta samassa huutaa lonkutti
räähkälintu, ronkkui nälkäinen korppi. Korpin kielellä vastasi paimen:
»täällä on ruokaa!» ja viisas, varovainen rosvolintukin tuli mitään
aavistamatta luo, saadakseen osansa saaliista — — ‒.

Yhä enemmän Pekko ihastui. Taipuisilla tuohillaan hän lintujen


vaihtelevia kieliä lasketteli, niillä hän lehdon laulajat pakinoille
pakoitti.

Kesän toisensa perästä Pekko sälyillään soitteli ja totteleva kalvo


muuttui satakieliseksi, tuhatääniseksi hänen huuliensa välissä.

You might also like